Retirement & Estate Planning Final exam

¡Supera tus tareas y exámenes ahora con Quizwiz!

The generation-skipping transfer tax (GSTT) lifetime exemption for 2021 is which of the following amounts?

The GSTT lifetime exemption amount for 2021 is $11,700,000.

Which of these is CORRECT regarding the generation-skipping transfer tax (GSTT)? An annual exclusion amount is allowed for lifetime direct skips. Gift splitting is allowed if both spouses elect it.

Both statements are correct.

Which one of the following statements regarding a qualified domestic relations order (QDRO) is correct?

A QDRO can identify the nonparticipant spouse as an alternate payee. A QDRO can identify the nonparticipant spouse as an alternate payee. A QDRO can identify a dependent as an alternate payee. A QDRO cannot force the plan administrator to increase benefits or provide a benefit option that the plan does not otherwise provide.

Your client established a funded revocable life insurance trust and transferred a $500,000 face value whole life insurance policy on his life into it. Your client's father was named trustee. The client has made no other lifetime gifts. If the client dies 18 months after establishing the trust, one disadvantage of this life insurance planning technique is that

For gift tax purposes, the value of a life insurance policy is the replacement cost, but for estate tax purposes, it is the face amount that is included and because death occurred within three years of the transfer, the face amount is brought back into the gross estate.

William dies on March 3, 2021, and is survived by his wife, Margie, and their three-year-old son, Max. For which of the following tax years is Margie eligible to file an income tax return using married filing jointly filing status? 2020 2021 2022

I and II Margie is eligible for married filing jointly status for 2020 and 2021. She may be eligible to elect qualifying widow status for 2022 if she provides a home for their qualifying dependent, Max.

Rhonda owns the following assets: A solely owned closely held business that comprises one-half of the value of her estate A collection of antique figurines that forms a substantial part of her remaining estate A residence owned with her husband as tenants by the entirety Rhonda's will bequests $10,000 to her only niece and leaves the balance of her estate to her husband if he survives her. Rhonda is looking for methods to provide the liquidity needed for her estate. Which one of the following actions would have the potential to improve the liquidity of Rhonda's estate?

None of the other actions will have any effect, positive or negative, on liquidity.

Ryan assumed that his interest in a motor home he owned with his brother, and that was titled as joint tenants with right of survivorship, would go to his son by a provision in Ryan's will. He was flabbergasted when a CFP® certificant informed him that if he died first, his interest would go to his brother, pursuant to the joint tenancy ownership rather than pursuant to his will. Which of the following is an example of a mistake, pitfall, or weakness?

Ryan mistakenly believes that the will would take precedence over how the motor home is titled when just the opposite is true. Improper disposition of assets

Which of the following charitable trusts permit additional contributions after the trust is initially funded? Pooled income fund Charitable remainder annuity trust (CRAT) Charitable remainder unitrust (CRUT)

Statement II is incorrect. A CRAT does not permit additional contributions after the trust is initially funded. Statements I and III are correct.

Jack created an irrevocable trust in 2014 for the benefit of his son, Bill, and his granddaughter, Karen. This year, the trustee distributes $40,000 in trust income to Karen. For purposes of the generation-skipping transfer tax (GSTT) the $40,000 distribution to Karen is a

This is a taxable distribution because it is a distribution made from a trust to a related beneficiary two or more generations below the transferor.

Jorge is in the highest current marginal income tax bracket. He owns several thousand shares of rapidly appreciating growth stock that he wants to transfer to his three minor children. Income will not be used for legal support obligations. He wants to have some control over the distribution of income from this stock while affording himself the gift tax annual exclusion for the total value of this and subsequent transfers to the maximum amount allowed. He also wants this stock to be available to his children when they reach age 21. Jorge does not want income from the stock to be taxed to him. Which one of the following is the most appropriate lifetime transfer technique for Jorge to use to best achieve his objectives?

A 2503(c) trust accomplishes all of Jorge's goals: maintaining control until the children reach 21, getting the annual gift tax exclusion, and giving the corpus to the children at age 21. Outright gifts cannot do all of that. A 2503(b) trust does not require the corpus to be distributed at age 21 and net gifts leave gift tax payable by the children which is definitely not one of Jorge's goals.

Which one of the following is a CORRECT statement regarding the advantages and disadvantages of using a conservatorship to manage property left to a child?

A conservator has a narrower range of powers than a trustee and the actions most definitely are supervised by a court. A guardian manages the day-to-day personal care of a minor while a conservator manages the finances.

Jud and Harry are equal partners in a small but thriving business. They recently signed a cross-purchase buy-sell agreement. They wish to use life insurance to fund their respective obligations in this agreement but want to do so with the least possible cost. Which one of the following types of insurance products is the most appropriate for Jud and Harry's situation?

A first-to-die policy will be less expensive than two individual policies and pays out upon the first death essentially ending the policy when it is no longer needed.

Your client is the sole depositor of $30,000 in a bank account. He is considering naming his girlfriend as the other joint tenant on the account. You should inform him that one consequence of creating the joint tenancy with right of survivorship bank account would be that

A joint tenancy with right of survivorship (JTWROS) designation is a will substitute that will allow this asset to pass outside of probate. The girlfriend, once added, can make withdrawals without his consent. The account value will be included in his gross estate and, since the girlfriend did not contribute to the account, any withdrawals she makes are gifts.

Bill is the sole owner of an Iowa farm valued at $1.63 million (farmland he owns is $1.04 million of that value). He has successfully farmed the land for the past seven years. There is no debt against any of the farm property. His will leaves the farm to his cousin, Charles, with his remaining property passing outright to his spouse, Mary; his godchild, Josh, is the contingent beneficiary of the residuary estate. Charles lives out of state and most likely will not continue the farming operation. Mary is named as Bill's personal representative. His gross estate is estimated at $4.1 million, with unsecured debts and administrative expenses projected to be no more than $100,000. Bill has made $1 million in adjusted taxable gifts. Mary recently received an inheritance of $400,000 from her mother's estate, most of which is rapidly appreciating commercial property. Before receiving the inheritance, Mary had a gross estate of $9.6 million and had made no adjusted taxable gifts. If Bill dies first, which of the following is the most appropriate postmortem planning technique for Bill and Mary to use to minimize estate taxes due in both estates?

A qualified disclaimer of part of Bill's property by Mary This is the correct answer because a qualified disclaimer by Mary of part of her bequest from Bill will prevent the disclaimed property from being subject to estate tax in Mary's estate because it would go to the contingent beneficiary, Josh. A partial disclaimer would more closely achieve equalization of the two estates, thus lowering overall estate taxes. The part disclaimed should allow Bill's estate to more fully use his estate tax credit amount.

When Marsha dies, her will leaves her entire estate to her surviving spouse, Daniel. Marsha's will provides that if Daniel does not survive Marsha by 90 days the estate will pass to their adult children. Daniel has sufficient assets of his own and would prefer that Marsha's property pass directly to their children now, instead of to him, but he wants to avoid making a taxable gift to them. Which of the following postmortem estate planning techniques would enable Daniel to achieve his objective?

A qualified disclaimer will achieve Daniel's objective because it constitutes a refusal by Daniel to accept the property and allows the property to pass directly to her children without any gift tax consequences. A partial QTIP election is used to qualify a terminable interest for the marital deduction, and a reverse QTIP election is used to permit the decedent's estate to use the generation-skipping transfer tax exemption for QTIP property. An election against the will allows a surviving spouse who has been disinherited under the deceased spouse's will to claim a share of the deceased spouse's estate.

With respect to the generation-skipping transfer tax (GSTT), which one of the following is an election that allows the donor or decedent's estate to be deemed the transferor of property that qualifies as an indirect skip?

A reverse QTIP election is used to pretend that the regular QTIP election was not made, thus allowing the donor or decedent's estate to be deemed the transferor of the property with respect to the GSTT.

For purposes of the generation-skipping transfer tax (GSTT) all of the following are considered skip persons except

A transferor's spouse or ex-spouse is not a skip person, regardless of age. Also, any children of a spouse or ex-spouse are only one generation below the spouse(s). For example, if a 70-year-old married a 30-year-old who already has a 3-year-old child from a different relationship, the 30-year-old spouse is legally considered to be of the same generation as the 70-year-old and the stepchild is only one generation below the 70-year-old.

Joe made a gift of property to his niece in the amount of $50,000 on the condition that his niece pay any gift tax due. Joe has previously made prior taxable gifts in the amount of the applicable exclusion amount. His niece, however, has never made a taxable gift. Which of the following are CORRECT statements about the tax implications of making this gift? The net amount of the gift (value of the gift minus gift taxes paid) minus the maximum annual exclusion amount will be included in Joe's estate tax calculation as an adjusted taxable gift. Joe will have taxable income to the extent that the gift tax paid by his niece exceeds Joe's adjusted basis in the property. Joe's niece will have a basis in the property equal to Joe's basis in the property as adjusted for gift taxes paid by her. If Joe's niece accepts this gift, she will have to pay a gift tax out of pocket.

All of these statements are correct statements regarding the tax implications of making this gift.

Which of the following statements regarding cross-purchase buy-sell agreements funded with life insurance is CORRECT? The death benefits under the life insurance policies are generally received tax free. The increase in the basis of the surviving owner(s) in the purchased interest(s) will equal the purchase price paid. The number of policies required may become cumbersome if there are a large number of businessowners.

All of these statements are correct. With a cross-purchase agreement, the surviving owners receive the life insurance death benefits income tax free. They then personally pay that amount to the former owner's family. This adds to the basis for the surviving owner(s). With an entity plan, the business itself receives the death benefits and then buys out the deceased owner's share of the business. The surviving owner(s)' basis is not changed.

Which of the following correctly explains why a beneficiary who received nonprobate assets by will substitute may agree to the use of all or part of the assets to meet estate liquidity needs?

All three answers correctly explain why a beneficiary who received nonprobate assets by will substitute may agree to the use of all or part of the assets to meet estate liquidity needs. First, without the aid of such nonprobate beneficiaries, the personal representative (PR) may be forced to liquidate probate assets to meet cash needs. If the nonprobate beneficiary also is to receive a bequest, it may be considerably reduced or eliminated because in a forced liquidation of estate assets, the estate is likely to have to sell assets far below their fair market value, and administrative expenses will be greatly increased. Second, if taxes are equitably apportioned, the PR usually has authority to withhold the probate share of a beneficiary who has received a nonprobate asset that generates estate tax or to sue such beneficiary for her share of the taxes. Finally, if property received by a nonprobate beneficiary was included in the decedent's gross estate, and estate taxes are not paid, the IRS has the right to pursue nonprobate beneficiaries to collect such tax. In some cases, this might involve the filing of a lien against the property that the beneficiary received from the decedent.

Joaquim is contemplating the sale of his solely owned business to his son in the form of a private annuity transaction. Which one of the following statements is CORRECT regarding the disadvantages of this type of transaction?

Annuities, even private ones, are based on life expectancy and if he does not reach that age, the full value will not be paid out. Additionally, if Joaquim lives beyond life expectancy, he will continue to be paid. Private annuities are based on FMV so they would not be subject to Chapter 14 rules. The payment of a premium for the right to cease making payments whenever his father dies applies to self-cancelling installment notes (SCINs), not private annuities.

Pele LaTerneau, who lives in a common law state, would like to make his brothers and sister part owners of the large farm holdings that he has acquired. Because the holdings operate best as a unit, he would like to keep the individual holdings together. His siblings have different needs, so he would like to give them variable amounts: 10% to Ramon, 25% to Felix, and 18% to Juanita. Given Pele's goals and situation, what is the most appropriate way for him to title the farm holdings for his siblings?

As tenants in common is correct.

Tasha and her mother, Marleen, meet with a financial advisor to get a better understanding of Marleen's estate and financial planning. Marleen has recently updated her will but Tasha fears there may be gaps in her mother's planning. If Marleen is in poor health, which of the following estate planning devices should Marleen include in her overall estate planning? Springing durable power of attorney for health care Advance medical directive Special needs trust Qualified domestic trust (QDOT)

Because Marleen is in poor health, there may soon be a need for someone else to step in and manage both her health care and her assets, making springing powers of attorney for health care and property desirable. She should also execute an advance medical directive (living will) so her physicians are informed of her wishes regarding end-of-life care. A qualified domestic trust (QDOT) is not appropriate as Marleen does not have a nonresident alien spouse.

Jon and Bob own a house as tenants in common. Jon owns 30% and Bob owns 70%. They purchased the house 10 years ago for $125,000 and today the house is valued at $200,000. If Jon dies today, what amount is included in Jon's gross estate?

Because the property is owned as tenants in common, the amount included in the gross estate is based on Jon's share of the fair market value on the date of death. Therefore, the value included in Jon's gross estate is $60,000 (30% of $200,000).

Sam and Sue paid $100,000 for their home five years ago. Its fair market value was $150,000 when Sam died. What was Sue's basis in the home after Sam's death if the home was held as community property?

Both halves of community property receive a stepped-up basis equal to the fair market value at the death of the first spouse.

Which of the following statements regarding income in respect of a decedent (IRD) is CORRECT? IRD is considered an asset for estate tax purposes and is reported on IRS Form 706. IRD is income for income tax purposes and is reported on either Form 1040 or Form 1041.

Both of these statements are correct. IRD is reported on Form 1041 if it is payable to the decedent's estate and on Form 1040 if it is payable to the decedent or the decedent's beneficiary. It is also part of the gross estate. There is nothing unique in an asset being subject to income tax and also being subject to estate tax. For example, dividends from a stock received in the year of death, but prior to death are subject to personal income taxes. If the dividends are not spent before death, they are also part of the gross estate.

Which of the following statements regarding a springing power of attorney is CORRECT? With a springing power of attorney, the attorney-in-fact's authority to act is delayed until the principal actually becomes incapacitated or incompetent. A springing power of attorney can be used in planning for the principal's possible incapacity.

Both of these statements correctly define a springing power of attorney.

Which of the following describe possible consequences of failing to consider business planning as part of the estate planning process? The illiquidity of the business may delay the completion of the administration of the estate. The absence of a buy-sell agreement creates the risk of the business interest being inherited by or sold to parties unfamiliar with the business.

Both statements are possible consequences of failing to provide business planning. A business interest requires special attention and handling.

Which of the following are reasons business succession planning is complex and challenging? Determining the value of the business may be difficult. It may be difficult to find a buyer who has the resources necessary to purchase the business. Family issues may be involved if the business is closely-held or family owned. A new owner may have difficulty adapting to the idiosyncrasies of the owner's business.

Business succession planning is complex and challenging for all of these reasons.

Which of the following statements regarding IRS Form 1041 is NOT correct?

Form 1041 for an estate must be filed within nine months following the decedent's date of death. Form 1041 for an estate must be filed on or before the 15th day of the fourth month following the end of the taxable year.

Greg died in 2021 and was survived by his wife and five children. At the time of his death, he owned the following property interests: Solely owned property valued at $16,000,000 Property owned in joint tenancy with right of survivorship (JTWROS) with his spouse, with his share valued at $1,000,000 Greg's will made no charitable bequests and provided that his entire estate go equally to his surviving children due to his wife having a large estate of her own. Other pertinent facts are: Greg made $1,000,000 in post-1976 taxable gifts. Greg's estate had $350,000 in allowable debts. Greg's estate had funeral and administrative expenses of $150,000. Greg's estate paid $60,000 in state death taxes. Which one of the following amounts most closely approximates Greg's net federal estate tax due? Use the Unified Federal Estate and Gift Tax Rates table.

Gross estate is $17,000,000. Subtract $560,000 in debts, expenses, and taxes to get $16,440,000. The marital deduction can be used for the JTWROS property, so subtract $1,000,000. Then you have to add $1,000,000 for adjusted taxable gifts, leaving the tax base at $16,440,000. Tax on this number is $6,521,800 [($15,440,000 × 40%) + $345,800]. Subtracting the applicable credit amount of $4,625,800 from $6,521,800 equals $1,626,000.

Which of the following statements regarding state property law elections and allowances is CORRECT? Family settlement agreements may require court approval. An election against the will can be made by the deceased's surviving children. A homestead exemption prevents surviving family members from losing certain property because of the claim of an unsecured creditor.

I and III Statement II is incorrect. An election against the will protects a surviving spouse, not the surviving children, from potentially being disinherited.

Which of the following would qualify as common estate planning goals associated with the taxation of capital gains? Delaying or avoiding the realization of a capital gain Timing an event that will cause the realization of gain to be short term Selecting an asset that will have the highest fair market value Designing the transaction so that the gain will be realized by the original owner

I only Common estate planning goals associated with the taxation of capital gains are delaying or avoiding the realization of a capital gain; timing an event that will cause the realization of gain to be long term; selecting an asset that will generate the smallest gain, either because of the asset having a high basis or no significant increase in value; and shaping the transaction so that the gain will be realized by a taxpayer who is in a lower marginal income tax bracket than the original owner.

Lewis is evaluating the assets in his estate to determine his liquidity. He owns assets such as collectibles, rental property, and closely-held business interests. Which premortem liquidity planning technique(s) would benefit Lewis? Lewis could reduce or eliminate the collectibles from his estate to enhance liquidity. Lewis could reduce or eliminate the rental property from his estate to enhance liquidity. Lewis could reduce or eliminate the closely held business interests from his estate to enhance liquidity.

I, II, and III Reducing or eliminating assets, such as rental property, collectibles, and closely held business interests, prior to death can enhance an estate's liquidity position because these assets are inherently more difficult to value and require more time and effort to administer. Thus, these assets are likely to generate higher administration expenses than other property, increase the liquidity requirements of an estate, and lead to estate shrinkage.

Which of the following is statements regarding a buy-sell agreement is CORRECT? It can fix the value for a business for estate tax purposes. It guarantees a market for the business after the business owner's or shareholder's death. If the potential buyers do not have sufficient liquidity to purchase the other owner's interests, then cash value life insurance on the owners can fund the agreement. It does not provide liquidity for the deceased owner's estate.

I, II, and III Statement IV is incorrect. When the deceased owner's business interest is purchased from the estate, the cash provides liquidity to the estate.

Which of the following assets generally receive a stepped-up basis at the transferor's death? The decedent's interest in property held jointly with right of survivorship Real estate Artwork Variable annuities

I, II, and III Variable annuities are not eligible for a stepped-up basis because they are income in respect of a decedent (IRD).

Roger Gomez has just learned that he has an inoperable brain tumor and has less than one year to live. He has come to you to get his financial affairs in order. Which of the following would be appropriate recommendations to make to Roger? He should establish a grantor-retained income trust (GRIT) with a five-year term, and transfer all of his assets into it to reduce his gross estate and avoid probate. He should put those assets that he wants to leave to a specific individual into an appropriate form of will substitute, if possible. He should consider executing a living will and/or a medical durable power of attorney. He should consider making a reverse gift of any highly appreciated assets that he has.

II and III Assets placed in a will substitute form can be enjoyed by the recipient at an earlier date than if they had to go through probate, and such assets will also lower administrative costs. He should make a living will and/or a medical durable power of attorney so that his wishes regarding extraordinary medical measures to keep him alive will be known and can be carried out should he become incompetent to voice such wishes or make such decisions at the end of his life. Assets placed in the GRIT would still be included in his gross estate if he died during the five-year term. Making a reverse gift would accomplish its main purpose, which is to eliminate gain in the asset by receiving a stepped-up basis from the deceased donee, only if the donee lives for more than one year after the gift (and Roger outlives the donee). The purpose of eliminating the gain is so that the asset can be sold at little or no tax cost. It is only after such sale that liquidity has been improved. It is doubtful that Roger will live long enough to get the desired step-up in basis, let alone accomplish the subsequent sale.

A CFP® professional is working with a client to develop the client's estate plan. As part of this process, the CFP® professional and client are attempting to project the estate's cash flow needs and develop a plan to ensure the estate's liquidity. The client owns several life insurance policies on his own life and expects his estate will be subject to estate tax. Which of the following statements regarding the development of the cash flow plan to maintain the liquidity of the client's estate is NOT correct?

It is generally not possible to reduce the estate's cash needs for estate tax. It may be possible to reduce the estate's cash needs by gifting the life insurance policies to get them out of the gross estate. If effective, this will lower the cash needs by the estate tax on the death benefits. To be effective, the donor, who owns the policies and is also the insured, must gift the policies more than three years before death. This is a situation in which an irrevocable life insurance trust (ILIT) would serve well. The death benefits would be out of the gross estate (assuming the transfer was more than three years old). The trustee of the trust might choose to use the life insurance death benefits received by the ILIT to pay the estate taxes. The trustee could also give or lend the money to the deceased's family to provide liquidity for debts, taxes, and administrative expenses. Any remaining assets in the ILIT would be distributed to the trust beneficiaries.

Frank is a widower. He has a $17.2 million estate consisting primarily of undeveloped real estate and life insurance. His children are the beneficiaries of his life insurance. His will leaves $900,000 of probate assets to each of his three children, with the residue to his cousin, James. Frank learned that his estate may have liquidity problems when he dies. Which one of the following techniques is the most appropriate to increase liquidity in Frank's estate?

Neither a QTIP trust nor an estate trust will change the liquidity situation. Naming the estate as beneficiary would cause that asset to go through probate and doesn't change the dollar amount that the children will receive.

Which of the following persons are not typically given priority in state statutes regarding health care proxies?

Priority for appointment as a health care proxy is given to a patient's legally married spouse, if any, and then to the patient's blood relatives, especially if they are the attorney-in-fact for a DPOAHC. A patient's unmarried cohabitant

Which one of the following is not a good technique to use to secure the payment of child support by a noncustodial parent?

Since the trust is revocable, the noncustodial parent could revoke the trust and remove the assets that are the surety for the child support payments. This could not be done if the trust were irrevocable. An irrevocable living trust is a good technique to use to secure the payment of child support by a noncustodial parent. An insurance policy for the benefit of the child is a good technique to use to secure the payment of child support by a deceased noncustodial parent.

Which of the following are CORRECT statements about the nontax characteristics of a pooled income fund? It involves a trust created by the grantor solely for the benefit of that grantor and the charity. It must pay a fixed dollar amount to the noncharitable beneficiary annually. The principal is distributed to the charitable beneficiary at the end of the noncharitable beneficiary's life. The income that can be paid to the noncharitable beneficiary is limited to 10% of the original principal unless the beneficiary is older than a specified age.

Statement I is false because a pooled income fund is not a grantor trust. Statement II is false because income payments are based on returns of the fund in proportion to the donor's contribution relative to the total value of the fund. Statement IV is false because there is no limitation on the income percentage. A pooled income fund is like owning a mutual fund as joints tenants with right of survivorship (JTWROS) with a charity along with many other co-owners who all get a share of the income.

Which of the following are characteristics of the probate process? It facilitates the administration of an estate without publicity. It provides for distribution of property through a judicially supervised process. It results in reduced administrative costs and expenses to the estate. It establishes a method for an orderly filing and paying of creditor claims against the estate.

Statement I is false because probate is a public, not private, process. Statement III is false because probate almost always results in increased administrative costs and expenses to the estate.

Which of the following characteristics are required for a power of appointment trust ("A" trust) to qualify for the marital deduction? Authorization to the trustee to split trust income between the surviving spouse and other family members Mandatory distribution of all income earned by the trust to the surviving spouse at least annually Control by the surviving spouse over the ultimate disposition of the trust assets Inclusion of the trust corpus in the surviving spouse's gross estate

Statement I is false because the spouse must be the sole income recipient in a marital trust, which is what a power of appointment trust ("A" trust) is.

Which of the following statements regarding property owned as joint tenants with right of survivorship (JTWROS) between spouses is CORRECT? The entire value of the property is included in the gross estate of the decedent spouse and the entire value of the property receives a stepped-up basis. One-half of the property is included in the gross estate of the decedent spouse and one-half of the property receives a stepped-up basis.

Statement I is incorrect because only one-half of the property is included in the gross estate of the decedent spouse and only one-half of the property receives a stepped-up basis. The stepped-up basis from the deceased spouse's half is added to the surviving spouse's original basis to determine the surviving spouse's new basis. This is different from community property in which both the deceased spouse and the surviving spouse receive a stepped-up basis.

Which of the following statements regarding cross-purchase buy-sell agreements funded with life insurance is CORRECT? The death benefits under the life insurance policies are generally subject to income tax. The increase of the basis of the surviving owner(s) in the purchased interest(s) will equal the purchase price paid under the cross-purchase buy-sell agreement. The number of policies required may become cumbersome as the number of businessowners increases.

Statement I is incorrect because the death benefits received under the life insurance policies are generally not subject to income tax.

Which of the following statements regarding the group term life insurance provided by an employer are CORRECT? The employer can deduct the premiums paid as a necessary and reasonable business expense. No part of the premiums that the employer pays will be considered income to any of the employees. The death benefit of the policy will be included in the employee's gross estate. The beneficiary of the death benefit will have taxable income to the extent that the death benefit exceeds the value of premiums paid by the employer.

Statement II is false because group life premiums for amounts over $50,000 are considered income to the employee. Statement IV is false because death benefits would be received income-tax-free.

If included as part of your married client's gross estate, which of the following property interests qualify for the marital deduction? A life estate interest in, and a general power of appointment over, the family residence (titled in his name only) to his wife A trust with income distributable at least annually to his wife and his children, with the remainder to the children at his wife's death A life income interest in a testamentary charitable remainder trust to his wife as the only noncharitable beneficiary A stock portfolio owned in joint tenancy with right of survivorship by the client and his brother

Statement II is false because income is also paid to the children and to qualify for the marital deduction, income must only be payable to the spouse. Statement IV is false because JTWROS property owned with his brother will not qualify for the marital deduction. The spouse is completely left out of that property, so no deduction is available.

Which of the following are characteristics of the probate process? It provides for the orderly distribution of property that passes by will or intestate succession to the ultimate beneficiary. It usually provides a longer time period for the filing of claims than if assets were to pass outside of probate. It provides for systematic administration of the decedent's estate. It provides for administration of all of the decedent's gross estate.

Statement II is false because there is a shorter, not longer, time period for the filing of claims than if assets were to pass outside of probate. Statement IV is false because gross estate is used only for estate tax purposes and the probate process has nothing to do with that calculation.

Which of the following regarding state property law elections and allowances are CORRECT? Family settlement agreements may require court approval. An election against the will can be made by the deceased's surviving children. A homestead exemption prevents surviving family members of the decedent from losing certain property due to the claim of an unsecured creditor.

Statement II is incorrect. An election against the will protects a surviving spouse, not the surviving children, from potentially being disinherited.

Which of the following estate planning techniques can be used by unmarried cohabitants to reduce estate tax due at the death of the first cohabitant to die? The gift tax annual exclusion The estate tax charitable deduction A qualified domestic trust (QDOT) A qualified personal residence trust (QPRT)

Statement III is false because a QDOT is used only when a U.S. citizen spouse wishes to leave assets to a nonresident alien spouse.

Which of the following are CORRECT statements regarding the characteristics and purpose of a "no contest" clause? This type of clause is used when a close family member is disinherited in a will. This type of clause imposes a penalty for contesting the validity of provisions in a will. This type of clause prohibits contesting the validity of provisions in a will. This type of clause prevents a person from contesting a will if he or she has feloniously caused the death of the testator.

Statement III is false because a no contest clause is designed to discourage a will contest, but cannot prevent it. A person who has feloniously caused the death of the testator is prohibited from collecting from the will by statute, which has nothing to do with a no contest clause.

Which of the following correctly identify a premortem technique that can be used to reduce the cash needs of an estate? Retitling property as joint tenants with right of survivorship (JTWROS). Executing a will that includes a self-proving clause and meets all legal formalities required by state law. Investing in real estate, such as rental properties, and retaining closely held business interests. Purchasing real property in multiple states for investment purposes. A)

Statement III is false because real estate and closely held businesses can only help increase cash available, but won't reduce cash needs. These will increase cash needs if liquid assets are used to acquire these assets. Statement IV is false because this will require ancillary probate which will increase cash needs, not decrease cash needs.

Which of the following features apply to both the federal gift tax model and the federal estate tax model? Unlimited marital deduction for qualifying transfers Unlimited charitable deduction for qualifying transfers Use of an applicable credit amount Allowance of an annual exclusion

Statement IV is incorrect because only the gift tax model allows for an annual exclusion.

In which of the following situations is a qualified domestic trust (QDOT) necessary for the donor to receive an unlimited gift tax marital deduction? The donor is a resident alien and the donee spouse is also a resident alien. The donor is a resident U.S. citizen and the donee spouse is a resident alien. The donor is a nonresident U.S. citizen and the donee spouse is also a nonresident U.S. citizen. The donor is a nonresident alien and the donee spouse is a resident U.S. citizen.

Statements I and IV are false because the donor needs to be a U.S. citizen.

Jacob and Wendy have been married for nine years and live in a common law state. They have two children, ages 4 and 2. In the same year they were married, Wendy insisted that they should each have wills. Since they had no children at the time, the wills they executed gave everything to the survivor, or if there was no survivor, to that person's brothers and sisters. Recently, the state in which Jacob and Wendy live has passed a statute allowing wills to be self-proved if they include language specified in the statute. Which of the following correctly state why Jacob and Wendy's current wills do or do not need to be amended? Their wills need to be amended to provide for their minor children's personal care if there is no survivor. Their wills do not need to be amended to provide for the children because of intestacy statutes. Their wills do not need to be amended to be self-proving, as they will be grandfathered since they were already validly executed. Their wills need to be amended to provide for the financial care of their children if there is no survivor.

Statements II and III are false because the wills should be amended to account for the new language required for self-proving wills.

Section 403(b) plan (tax-sheltered annuity plan or TSA) employer contributions must abide by the annual additions limit. must not discriminate in favor of highly compensated employees. are based on a maximum annual covered compensation of $230,000 in 2020. are subject to FICA (Social Security and Medicare) and FUTA (federal unemployment) payroll taxes.

The answer I and II. A Section 403(b) or TSA plan is subject to the annual additions limit of the lesser of 100% of compensation or $57,000 for 2020. Contributions must not discriminate in favor of highly compensated employees. Employer contributions are based on a maximum annual covered compensation of $230,000 and are not subject to FICA and FUTA payroll taxes (employee deferrals are subject to FICA and FUTA).

Jack is a single taxpayer who retired at age 62 and receives a qualified plan pension of $1,500 each month. He has begun working as a consultant to various firms and is projecting he will earn $70,000 in 2020. What is the maximum deductible contribution Jack may make to a traditional IRA for 2020?

The answer is $7,000. Jack is not currently an active participant in a qualified plan, is age 50 or older, under 70½, and has earned income in 2020. Jack may make a deductible IRA contribution of $7,000 (6,000 + $1,000 catch-up) for 2020.

Several of the steps involved in the estate planning process are identify and select goals, monitoring progress and updating, develop the appropriate technique, and analyze potential alternate courses of action. Which one of the following lists the sequence of these steps correctly?

The answer is I, IV, III, II. Understanding the client's personal and financial circumstances must be completed prior to identifying and selecting goals. From there, you can analyze the client's current course of action and potential alternative courses of action. After analysis, you can develop recommendation(s) and present them to the client. With client agreement, you can help implement recommendation(s) and, of course monitor and update as needed.

A qualified plan is a company-sponsored retirement plan with benefits guaranteed by the Employee Retirement Income Security Act (ERISA). a tax-efficient way to save for retirement. only applicable for firms with 50 or more employees. considered a plan that benefits highly compensated employees only.

The answer is II only. ERISA does not guarantee plan benefits; the Pension Benefit Guaranty Corporation (PBGC) guarantees benefits in defined benefit pension plans. A qualified plan is a tax-efficient way to save for retirement. Qualified plans may be established for firms with as few as one employee. Qualified plans also benefit non-highly compensated employees.

Which of the following qualified plan distributions is subject to the 10% penalty for early withdrawal?

The answer is an in-service hardship distribution from a Section 401(k) plan to an employee-participant, age 55. Even if the distribution is a hardship withdrawal, the penalty applies unless the employee-participant has attained the age of 59½ (or one of the other 10% penalty exceptions applies).

Andy, age 68, has a gross estate currently valued at $2,500,000 that consists primarily of highly appreciated growth securities. Within the last six months, Andy transferred $500,000 worth of these securities to his wife, Harriet. His cost in these securities was $200,000. Harriet recently died. The fair market value of the transferred securities at the time of her death was $500,000. The securities passed to Andy under the terms of Harriet's will. Which one of the following is an income tax implication of the transfer of stock?

The basis Harriet received from the lifetime gift from Andy would be the same as Andy's. Because she did not live for more than one year, the property does not qualify as a reverse gift and get a stepped-up basis. The basis will remain $200,000.

Jose and Maria have been married for 50 years. They have three children and seven grandchildren. Their estate is $30 million. They do not want to rely on the deceased spouse's unused exemption (DSUE) because they have seen a lot of tax law changes through the years. They would like to arrange their assets so there are no estate taxes when the first spouse dies. Maria wants to be sure their son, Fred, receives their vacation home when the second of them passes away. Jose wants to ensure their daughter Amelia eventually takes possession of their family home upon the second of the couple to die. Other than these assets, they are fine with allowing the survivor to control the remaining property. Which of the following will best allow them to accomplish all three goals?

The combination of the two is the only way to achieve all of their goals as the remaining choices except for the QTIP will allow for some of the goals to be achieved, but not all of them. The QTIP trust doesn't align with their goals at all.

Ruth established a Section 2503(c) minor's trust for her 5-year-old granddaughter, Frances, and funded it with $20,000 of income-producing real estate. Ruth named herself as trustee. Which one of the following statements is CORRECT regarding this transfer?

The corpus of 2503(c) trusts must be distributed by age 21. As long as Ruth has control of trust distributions, it remains in her estate. A 2503(c) trust can be funded with real estate. Income-producing real estate would be a good way to generate the income desired by such a trust.

Your client died recently with a gross estate valued at $425,000. Her estate tax bracket is 34%. Her husband has a gross estate of $155,000. Her will (1) placed $350,000 in a trust that gave her husband and her mother life income interests payable annually, with the trust remainder going to her son from a previous marriage when both have died; and (2) left the residue to her daughter. Her husband was named executor. Most of her estate is in certificates of deposit and securities that recently have shown only minimal growth. During the last six months of her life, she had uninsured medical expenses of $30,000. She and her husband were in the 24% marginal income tax bracket at the time of her death. If her husband, as executor, came to you for advice, you should inform him that the most appropriate postmortem election for his wife's estate is

The estate is way too small to need a QTIP election. Waiving executor fees will help, but not as much as claiming the medical expenses on her final income tax return. With clearly no chance of owing estate taxes, the AVD is not an option.

Ten years ago, a donor gifted property worth $100,000 but retained a life estate in the property. The donor died last month when the property's fair market value was $500,000. What amount will be included in the donor's gross estate?

The full fair market value of the property at death will be included in the donor's gross estate because the donor retained a life estate in the property. 500,000

Assuming that a decedent left no valid last will and testament, which of the following assets will pass by the laws of intestate succession?

This asset remains after his death and does not pass by will substitute as the remaining answer choices do.

Which of the following are correct characteristics or results of leveraging the generation-skipping transfer tax (GSTT) exemption? Allocation of the GSTT exemption is made on a timely filed gift tax return that reports the transfer. The allocation becomes effective as of the date of the transfer. No future additions are made to the trust. All future taxable distributions and terminations will be free of GSTT.

This is the best answer, as all listed options correctly state characteristics or results of leveraging the GSTT exemption.

Your client, age 65, has a gross estate valued at $18.7 million. He and his second wife have two teenage children. In addition, your client has two children from his first marriage who are in their mid-30s. His objectives are to: leave an income stream and a portion of his estate to his current wife; leave a portion of his estate in trust for the teenage children from his current marriage; ensure that the children from his first marriage receive a portion of his estate; and reduce his federal estate tax liability. Which one of the following transfers is most appropriate for achieving the client's objectives?

This is the only option that achieves all of the client's goals. The family bypass trust and the outright gift options leave out his children from his first marriage. There is no guarantee that whomever has the power of appointment in a power of appointment trust will leave anything to the other parties.

Which of the following are CORRECT statements concerning a buy-sell (business continuation) agreement funded with life insurance? The business is a party to the contract if a stock (entity) redemption plan is used. With a cross-purchase plan, the surviving shareholder's new cost basis is equivalent to his or her old cost basis plus the life insurance proceeds used to purchase the deceased shareholder's interest at the price established by the agreement. A cross-purchase plan is preferable to a stock (entity) redemption plan when all shareholders are in a higher income tax bracket than the corporation. With a stock (entity) redemption plan, premiums paid by the corporation on life insurance to fund the purchase are taxable income to the shareholders because they will eventually benefit. Under a stock (entity) redemption plan, the value of the deceased's business interest is included in his or her gross estate, while the life insurance proceeds used to purchase his or her business interest are excluded.

When shareholders are in a higher income tax bracket than the corporation, a stock (entity) redemption, not a cross-purchase plan, is preferable because payment of premiums by the corporation provides a greater economic benefit to the shareholders than payment of salaries and/or dividends to them which will be taxed at a higher rate before they are used to pay premiums. Under a stock redemption plan, the premiums are paid by the corporation with after-tax dollars (i.e., premiums are not deductible to the corporation) and therefore are not taxable income to the shareholders.

When is an employee stock ownership plan (ESOP) an appropriate choice for an employer to implement? The employer is either a C or an S corporation. Creating a market for the employer stock helps diversify the employer-owner's stock portfolio. The employer wishes to increase the company's liquidity by pledging the stock for a loan in the name of the ESOP. The employer wishes to transfer ownership of the business to the employees.

The answer is I, II, III, and IV. All these statements regarding ESOPs are correct.

Cisero gifted 2,000 shares of his stock in a closely held corporation to his daughter. These shares constitute half of the total number of shares he owned in the corporation prior to the transfer. The value of Cisero's stock in this corporation prior to the transfer was $400,000. Which one of the following statements is CORRECT regarding the value of the stock transferred to his daughter?

This transfer would not qualify for a blockage discount as it is not publicly traded stock. It would not qualify for special use valuation either, but a reasonable lack of marketability discount is available and would not violate Chapter 14 rules.

The estate planning tool that would allow an attorney-in-fact to expedite the principal's Medicaid eligibility, arrange for in-home or nursing home care, hire necessary health care personnel, or employ companions is a

A durable power of attorney allows the attorney-in-fact to make decisions for the principal during a period of incapacity.

Which of the following are factors that a financial planner should monitor for every client? Changes in the client's objectives Changes in the client's marital status Changes in property laws Changes in the amount of lifetime gifts made by the client

All of these are factors that a financial planner should monitor for every client.

Which of the following are factors that should be considered in selecting a trustee for a trust that will last for an extended period of time? Appointing of co-trustees Appointing a contingent trustee Providing a method for the appointment of a successor trustee The age of the potential trustee

All of these are factors that should be considered in selecting a trustee for a trust that will last for an extended period of time.

Which of the following are valid concerns when an applicant for long-term nursing home benefits under Medicaid transfers title to his or her personal residence to his or her community spouse? The possibility the parties may divorce Whom the community spouse may leave the residence to when he or she dies The community spouse's creditors The terms of an existing mortgage on the residence

All of these are valid concerns when considering Medicaid as an option to pay for long-term care.

Which of the following actions should an executor take when developing a cash flow plan for an estate? Determine which assets are the best candidates for sale if it appears necessary to sell estate property Begin planning a strategy to make up any projected cash deficit Account for the possibility of unexpected expenses Explore ways of reducing the estate's cash needs by making special elections under the estate tax laws

An executor should take all of these actions when developing a cash flow plan for an estate.

When she died, Roberta and her spouse, Patrick, owned a condo as joint tenants with right of survivorship (JTWROS). Their basis in the condo was $500,000, and the fair market value on the date of Roberta's death was $1,200,000. What is Patrick's basis in the home following Roberta's death?

Because Roberta and Patrick were spouses, Patrick receives a stepped-up basis of $600,000 in the 50% of the condo that is included in Roberta's gross estate. He retains his original basis of $250,000 in the 50% that is not included in Roberta's gross estate. Patrick's new basis is $850,000 ($250,000 + $600,000).

Which one of the following statements describes a basic feature of one of the special valuation rules under IRC Chapter 14?

Chapter 14 rules are best remembered as a set of "anti-shenanigans" laws requiring intrafamily transfers to meet certain restrictions. Unrelated partners would not fall under Chapter 14 rules.

Assuming that the special valuation rules under IRC Chapter 14 apply to each of the following situations, which one of the following statements is CORRECT?

Chapter 14 rules do not negate other regulations. A retained interest will reduce the value of the gift because the done is not receiving the entire gift. The retention of an annuity or unitrust interest by the grantor is deemed to be a qualified interest.

Which statement regarding tax planning techniques for unmarried cohabitants is NOT correct?

Converting solely owned property to tenancy in common property between unmarried cohabitants will ensure that the surviving cohabitant will eventually own the entire property. Since each tenant in common is entitled to dispose of his or her interest to anyone, there is no guarantee that the interest of the first cohabitant to die will pass to the surviving cohabitant. It definitely will not pass to the surviving cohabitant unless it is passed in the deceased tenant's will. JTWROS titling would accommodate this. Since the only requirement to be entitled to the annual exclusion is that the donee be given a present interest in the property, the donor will be entitled to an annual exclusion.

In which one of the following situations would a state's intestate succession laws be applied?

Intestacy applies when a person dies without a will or with a will that fails to include all probate property.

If Arthur died today with the following assets, which would be considered income in respect of a decedent (IRD)?

Distributions from Arthur's IRA made after the date of death. Income in respect of a decedent is income to which the decedent was entitled at the date of death but had not yet received. Distributions from the decedent's IRA made after the date of death are an example of IRD.

The premature distribution penalty does not apply to which of the following IRA distributions? A distribution paid to a beneficiary after the death of the IRA owner who had not begun receiving minimum distributions A distribution made after the owner is age 55 and after separation from service A distribution made for purpose of paying qualified higher education costs A) I and III B) III only C) I only D) I and II

Explanation The answer is I and III. A distribution made after the owner is age 55 and after separation from service is only an exception for distributions from qualified plans, not from IRAs.

Which one of the following statements regarding Henry, who recently married for the first time, is CORRECT?

In a community property state, Henry's earnings from his job subsequent to the date of his marriage will be considered community property. Income earned after marriage is considered community property.

Which of the following is a financial goal?

Maximizing benefits for a surviving spouse

Mike and Jane, a married couple, bought a condominium 15 years ago for $200,000 as joint tenants with right of survivorship (JTWROS). When Jane died, the condominium was valued at $500,000. Four years after Jane's death, Mike sells the condominium for $600,000. What is the amount of Mike's capital gain?

Mike received a stepped-up basis in one-half of the condominium at Jane's death. He retains his original basis of $100,000 in the other half. His total basis at the time of the sale is $350,000, so his gain is $250,000.

Stella transferred a residence that was in her sole name to her and her four children as joint tenants with right of survivorship to avoid probate of this asset. No child provided any consideration for this transfer. When Stella dies, what percentage of the fair market value of this asset must be included in her gross estate?

Only Stella contributed to the acquisition costs of the asset. Under the consideration furnished rule, there is a rebuttable presumption the deceased joint tenant's gross estate must include 100% of the property's value. Since Stella furnished all the consideration, there is 100% inclusion.

For purposes of the generation-skipping transfer tax, any payment of income or principal from a trust to a person two or more generations junior to the donor's generation is a

Payment of income or principal is a distribution, not a transfer or termination. A distribution might be to a direct skip, or it could be to an indirect skip.

Jill, Sherry, and Peggy are each one-third owners of a closely held business and they have executed a buy-sell agreement. The agreement requires that Jill, Sherry, and Peggy each purchase and pay the premiums on an insurance policy that insures each co-owner and names the policyowner as the beneficiary. Which of the following correctly states an advantage or disadvantage of this buy-sell agreement?

Premiums paid are not income or gifts and the death benefit is what is included in the owner's gross estate.

Which of the following is NOT a potential advantage of a buy-sell agreement funded with life insurance?

Premiums paid on the life insurance policies used to fund the agreement are tax deductible. The premiums paid on a life insurance policy used to fund a buy-sell agreement are not tax deductible.

Sam wants to leave some real estate to his wife, Inez, outside of probate when he dies. He would also like her to receive some interest in the real estate while he is alive. Sam does not want Inez to be able to transfer or encumber her interest in the real estate without his consent. Which one of the following is the most appropriate form of will substitute for Sam to use?

Tenants in common and JTWROS are partitionable without consent, which is not one of Sam's goals. Giving Inez a remainder interest does not give her the present interest he wants her to have while he is alive.

George has a gross estate valued at $3.7 million. His estate consists almost entirely of publicly held stock owned solely by him. He owes no debts. George's only living relative is a nephew whom he hasn't seen or heard from for 30 years. George has not executed a valid will. If George were to die in the current year without change in any of the related facts, which one of the following is a disadvantage of the probate process for him?

The marital deduction has nothing to do with probate. Payment of a personal representative's fee would reduce his estate tax, but his estate is small enough that his applicable credit amount will certainly cover any taxable estate. Probate requires court supervision regarding payment of claims and distribution.

Generally, for generation-skipping transfer tax (GSTT) purposes the measure of value is

The measure of value is generally the property's FMV.

An estate elects to value property using the alternate valuation date. Subsequently, some property in the estate is sold after the date of death and before the alternate valuation date. What is the valuation date for that property?

The property is valued as of the date of sale. If the estate elects to use the alternate valuation date and property is disposed of within the six-month period after the decedent's death, the valuation date is the date of disposition.

Which of the following is NOT a requirement of a qualified disclaimer for federal transfer tax purposes?

The refusal must be written and received by the donor, the donor's legal representative, or the legal titleholder of the property within six months of the decedent's death or six months from the date of gift or, if later, within six months after the beneficiary becomes 21 years of age. The time frame for notification of refusal is nine months rather than six months.

Which one of the following statements regarding qualified domestic trusts (QDOTs) is incorrect?

The trustee or trustees must all be U.S. citizens or corporations. Only one trustee must be a U.S. citizen or U.S. corporation. A power of appointment, QTIP, or CRAT or CRUT in which the grantor's spouse was the sole income beneficiary, would meet this requirement. Without this election, the unlimited marital deduction will not be granted.

Bob, Frank, Hector, and Fermin are equal partners in a closely held business. Although the four partners work well together, their spouses and children do not. No partner is ready to quit the business and retire, but they are each worried about how the business would operate if this were to happen. Each partner is financially overextended and thus not able to pay a gift tax or capital gains tax, as they started the business from scratch and it has become very profitable. Which one of the following is the most appropriate business transfer technique for the partners to use considering these circumstances?

This is the best choice because only four policies are required with an entity purchase plan as compared to the 12 policies a cross-purchase plan would require. Neither a private annuity nor a preferred stock recapitalization make sense for this scenario as the partners are not family members.

Darwin, age 60, has an estate valued at $15 million. Included in the valuation of his estate are the following: A small U.S. Treasury note that is subject to changes in market value One-half ownership in Coeur d'Alene Estates, a private real estate development owned as a joint tenant with rights of survivorship with his sister, Anne 800,000 shares of Untell Inc., a public corporation traded on a major exchange; 50,000 shares of this stock are traded daily A joint and last survivor annuity that names his daughter, Ruthie, as the surviving annuitant When Darwin dies, which one of the following valuation techniques would most effectively reduce the value of his gross estate, and why?

With 800,000 shares of stock in a company that only trades 50,000 shares daily, this is the rare example of when a blockage discount would be available.

Jane has a gross estate estimated at $18 million. Approximately 75% of her estate is attributable to the value of personal property and collectible items. Jane is married but has no children. Her husband does not have a large estate because he spends money freely and foolishly. Because she is much older than her husband, Jane would like for him to benefit from her wealth after her death without giving him control over the principal either while he is alive or at his death. Jane wants as little of her estate assets as possible to go toward payment of estate taxes on either of their estates. She currently has no will but has come to you for advice regarding provisions she should put in a will. Which provision, if placed in her will, would be best to increase the liquidity of her estate and accomplish her other goals?

A QTIP wouldn't change the liquidity situation. A power of appointment trust gives the husband access to trust principal she does not want him to have while she is alive and the testamentary trust allows the same access upon her death.

Janis owns the Pretty Little Celluloid Shop as a sole proprietor. Janis is now 63 years old and is ready to retire. She has a gross estate estimated at $3.9 million, and the value of the business constitutes $2.45 million of that amount. Janis would like to transfer the business to her daughter and remove all future appreciation of it from her estate. In addition, she would like to receive an income stream from the business for the rest of her life. Which one of the following is the most appropriate form of business transfer for Janis to use to best achieve her objectives?

An installment sale will only provide income for a period of time. The remaining two options don't generate income, but simply bring her daughter in as co-owner.

Paul and Cheryl are husband and wife who initially lived in a community property state. Soon after their marriage they began establishing an emergency fund using money that each earned from their respective jobs. This fund was used to meet unexpected expenses as they arose. Three years ago, Cheryl liquidated stock that she had purchased prior to her marriage, and placed the proceeds in the emergency fund. There have been many deposits and withdrawals from the fund since that time. Last year, Paul filed for divorce. Cheryl is seeking to recover the full value of the stock proceeds that she placed in the emergency fund as her sole and separate property, and half of the remaining emergency fund. Paul claims he is entitled to half of the entire emergency fund. Which one of the following statements is CORRECT regarding Paul's and Cheryl's rights in the emergency fund?

Because Cheryl commingled separate property with community property, the stock proceeds lost their separate property status and because community property.

Two sisters, Donna and Mary, own a home as joint tenants with right of survivorship (JTWROS). They purchased the home 10 years ago for $100,000. Donna contributed $40,000 and Mary contributed $60,000. Today, the home is valued at $180,000. If Donna died today, what amount would be included in her gross estate?

For JTWROS property owned by non-spouses, the amount included in the gross estate is based on the decedent's percentage of contribution to the original cost of the property. Donna contributed 40% of the purchase price of the home, so 40% of its value is included in her gross estate ($180,000 × 40% = $72,000).

When Leah dies, she owes $5,000 in unpaid medical expenses. Which of the following statements concerning the tax treatment of these expenses is CORRECT?

For the expenses to be deducted on Leah's final Form 1040, however, the deductions must be itemized and the expenses must meet the 10% of AGI floor.

Because of her financial stability and sizable net worth, Marleen Burke is intending to simply leave the funds in her IRA untouched. When she dies, she believes that this asset will get a step-up in basis for her heirs. Which of the following statements regarding Marleen's IRA is CORRECT? She must receive minimum distributions after attaining the age of 72, but any remaining amounts in the IRA at her death will receive a step-up in basis. She is correct in her belief, and this is a great strategy. The heirs will not receive a step-up in basis, and she will be penalized if she does not receive distributions when required by the Internal Revenue Code.

III only There are a couple of problems with Marleen's plan. She is required to take distributions from the IRA once she reaches her required beginning date. Retirement accounts are considered income in respect of a decedent (IRD) assets and do not receive a step-up in basis at the owner's death.

In a money purchase pension plan, forfeitures revert to the plan. may be used to reduce future employer contributions. can be reallocated among the remaining plan participants. do not count against remaining participants' annual additions limits.

The answer is I, II, and III. Forfeitures count against the remaining participants' annual additions limits.

Which of the following forms of property ownership provides a right of survivorship when an owner dies?

Tenancy by the entirety is a special form of joint tenancy with right of survivorship (JTWROS) that can exist only between spouses. None of the other answer choices provides a right of survivorship.

Richard participates in a traditional defined benefit pension plan at work. His projected monthly benefit under the plan is $1,000. If the plan provides life insurance for Richard, the death benefit payable under the policy is limited to

The answer is $100,000. Defined benefit plans use the 100 times test for determining whether they comply with the incidental benefit rules. Under this test, the death benefit cannot exceed 100 times the participant's projected monthly benefit (in this instance, $100,000).

Which of the following is a skip beneficiary for purposes of the generation-skipping transfer tax (GSTT)? A related person who is at least two generations below that of the transferor. A trust in which the beneficiaries are skip persons and from which no non-skip person will benefit. An unrelated person who is younger than the transferor by 37½ years or more.

These are all skip persons for purposes of the GSTT.

Which of the following are prerequisites for application of the generation-skipping transfer tax (GSTT)? A gratuitous completed transfer Transferee deemed to be two or more generations younger than the transferor Transfer qualifies as a direct skip transfer No exceptions or exemptions from the normal rules apply

These are the three prerequisites for application of the GSTT. Transfers can qualify as either a direct or indirect skip and be liable for the GSTT. The times at which the GSTT must be reported and will be due depends in the first instance on whether the GST is a direct or indirect skip.

A surviving spouse's interest in property that might terminate or fail upon the lapse of time or upon the occurrence or nonoccurrence of some contingency is known as a

This is a terminable interest.

Robert is the sole income beneficiary of a charitable remainder unitrust (CRUT) established by his recently deceased wife in her will. A qualified public charity will receive the trust remainder at Robert's death. Which one of the following is a CORRECT statement regarding the effect of this trust on the potential liquidity of Robert's estate at his death?

This is because upon his death the asset belongs to the charity creating neither a liability for the estate nor a potential source of income. It has simply been removed from the estate.

Which of the following is NOT an attainable estate liquidity planning strategy?

This is not an estate liquidity planning strategy because all estates, whether large or small, have administrative expenses. These expenses can be diminished by proper premortem planning but can never be completely eliminated. Eliminating all estate administrative expenses

Which of the following statements is CORRECT regarding a CFP® certificant's role in defining a client's financial goals, needs, and priorities? The role of the planner is to facilitate the goal-setting process. IThe role of the planner is to assist clients in recognizing the implications of unrealistic goals and objectives. The role of the planner is to make sure a client's goals and objectives are consistent with the client's values and attitudes. The role of the planner in this process will involve exploring a client's expectations and time horizons.

all

Your client has made the following lifetime cash gifts: Total Gifts to DaughterTotal Gifts to Son2018$35,000$75,0002021$90,000$55,000 How much of your client's applicable credit is gone on December 31, 2021?

Remember the cumulative nature of the gift tax and the annual exclusion amount for present value gifts. The total gifts to the daughter and the son in 2018 are reduced by the $15,000 annual exclusion to arrive at a net taxable gift total of $80,000 ($35,000 − $15,000 + $75,000 − $15,000). The same applies to the 2021 gifts resulting in a net gift totaling $115,000 ($90,000 − $15,000 + $55,000 − $15,000). To calculate the tax you need to first figure the tax on the total of the gifts from both 2018 and 2021 ($80,000 + $115,000 = $195,000). Tax on $195,000 would be $38,800 + 32% of $45,000 (the amount over $150,000), which is $14,400. $14,400 + $38,800 = $53,200.

To be eligible to adopt a SIMPLE 401(k), an employer may have no more than

The answer is 100 employees who earned at least $5,000 last year. An employer with 100 or fewer employees who earned $5,000 or more during the preceding year may adopt a SIMPLE 401(k).

What is the taxable character of distributions that are made from a Roth IRA?

The answer is tax-free income if the distribution meets the holding period and qualified distribution requirements. Distributions made from a Roth IRA are income tax free if the Roth IRA meets certain specified conditions. These conditions include meeting the 5-year holding period and 1 of the following: (1) the owner is age 59½ or older, (2) distribution upon disability of the owner, (3) distribution to a beneficiary upon the death of the owner, or (4) for a first-time home purchase (up to a $10,000 lifetime cap).

Lauren and Roger are spouses. Lauren has assets with a market value of $50 million titled in her name alone. Roger has assets valued at less than $1 million. Lauren drafts a will making an outright bequest of all of her assets to Roger. Which of the following are potential disadvantages of Lauren's approach? Lauren will not use her estate tax applicable exclusion amount when she dies. Roger may become legally incapacitated and not be able to manage the property. When Roger dies, the property must be included in Roger's gross estate to the extent Roger has not spent it or consumed it during his lifetime. The DSUE amount may not be available in some circumstances.

All of these are potential disadvantages of leaving all property outright to the surviving spouse.

Which of the following are the tax implications of a 10-year term charitable lead trust with the donor's children as the remainder beneficiaries? The donor's charitable gift tax deduction is determined by the present value of the charity's right to receive trust assets at the end of the 10-year term. The donor is liable for gift tax based on the entire value of the gift to the children as discounted to the date of the gift. The entire value of the assets gifted to the trust will be removed from the donor's gross estate only if he or she outlives the 10-year term. Each year, as the trust pays income to the charity, the donor receives a charitable income tax deduction for that amount.

In a charitable lead trust, the charity receives the income with the remainder to a noncharitable beneficiary. The present value of this remainder interest is taxable. Therefore, the charitable gift tax deduction I is the present value of the income interest. III is incorrect as the entire value of the trust assets is removed from the grantor's gross estate no matter when the grantor dies. IV is incorrect as the amount of the charitable income tax deduction is based on the value of the gift to the charity in the year the gift is made, not on income earned.

All of the following items of property would be considered community property in a community property state except

In a community property state, property acquired by one of the spouses by gift or inheritance during their marriage is considered to be separate property. Items owned prior to the marriage can also be separate property. real estate received by one spouse during marriage as an inheritance from her mother.

Rolando owned a parcel of real estate as an equal tenant in common (TIC) with his wife, Liz, and his brother, Sam. Rolando and Liz each contributed $50,000 to the original purchase price, and Sam contributed $20,000. Rolando recently died and is survived by Liz and Sam. Which of the following statements are CORRECT concerning a tax implication of this form of property ownership? Rolando's estate must include one-third of the property's fair market value (FMV) as of the date of death. When they took title as TIC, both Rolando and Liz made a gift to Sam. Rolando's estate must include 41.66% of the property's FMV at the date of death, unless his personal representative can prove contribution by Sam. After Rolando's death, Liz will be entitled to receive 83.33% of the income from the property because she will receive Rolando's interest by right of survivorship.

Rolando's estate must include one third of the date of death FMV of the property because that is his percentage share of ownership. Contribution by the parties would be relevant only if the property were owned in joint tenancy. Sam obtained a one-third ownership interest, but paid less than one-third of the purchase price; thus, Rolando and Liz each have made a gift to Sam. Liz will not automatically receive Rolando's interest, as tenancy in common has no right of survivorship feature.

Rollie plans on purchasing some U.S. savings bonds with his son, Steven. He has been told that he can title the bonds either as "Rollie or Steven" or "Rollie payable on death to Steven." Which of the following statements are CORRECT regarding advantages and disadvantages of these two methods of titling? "Rollie or Steven" would not avoid probate of the bonds. "Rollie payable on death to Steven" would give Rollie sole control of the bonds during his life. "Rollie payable on death to Steven" would allow Rollie to remove Steven as beneficiary. "Rollie or Steven" would allow the survivor to become the sole owner of the bonds without the bonds going through probate.

Rollie would avoid probate using a joint tenancy with right of survivorship (JTWROS) or payable on death (P.O.D.) designation because these are will substitutes.

Of the following actions taken last year by Joan, which transfers must be included in calculating her total gifts for last year? Purchase of a certificate of deposit (CD) that is payable to her daughter on Joan's death Writing a check to her mother for $3,600 to assist her in paying for recent surgery Placement of her brother's name jointly with her own on the deed to a commercial office building that she purchased Cancellation of an $25,000 debt owed to her by her best friend

Statement I is false because the gift has not been completed. The daughter only has a future interest in the CD.

Which of the following are important characteristics of the gift tax marital deduction? It enables the donor to avoid gift tax liability by transferring the entire liability for gift taxes to the donee spouse. It allows the donor to avoid gift tax liability on up to one-half of the value of the gifted property that is received by the donee spouse. It allows the donor to avoid gift tax liability on the amount of the gift in excess of the annual exclusion amount. It allows the donor to avoid gift tax liability on a gift to a donee spouse.

Statement I is false because there would be no gift tax liability for the spouse. The marital deduction would transfer estate tax liability, but not gift tax liability. Statement II is false because it allows the donor to avoid gift tax on 100% of the value.

Which of the following are CORRECT statements about the filing requirements and/or the responsibility for payment as they relate to federal transfer taxes? A donee can be held responsible for paying the gift tax on a transfer that she has received if the IRS cannot collect from the donor. A federal estate tax return need not be filed unless an estate owes estate taxes in excess of the unified credit. The beneficiary is responsible for paying the generation-skipping transfer tax on a distribution from a trust and must file a tax form. A federal gift tax return need not be filed for a gift that is split with the donor's spouse.

Statement II is false as an estate tax return is always required. Statement IV is false because the exact opposite is true: A gift tax return, while not always required, is required when gift splitting is used.

Your client has an estate valued at $3 million. Two months ago, his wife died. He and his deceased wife did not have any children together, but she had two children from a prior marriage. His will, drafted in 2007, leaves everything to his wife. Nocontingent beneficiary is named in the will, and it does not contain a residuary clause. Included in the client's estate are real estate holdings in three other states. He wants to retain lifetime ownership of these properties because of the income they provide him. He would like the real estate holdings to pass to his wife's children in equal shares upon his death. He would like the remainder of his estate to go to his brother. Which of the following are serious estate planning pitfalls that can be avoided if your client amends his will to carry out his objectives? Having the estate pass under the laws of intestacy Having the estate assets distributed through probate Having the estate pay any estate tax Having part of the estate pass to unintended beneficiaries

Statement II is false because amending a will has no effect on whether probate can be avoided. Wills are probated. Statement III is false because amending a will has no effect on the estate tax calculation.

Which of the following are characteristics of a gift-leaseback? The property involved in the transaction usually is a business-related asset. The property involved in the transaction usually is gifted to a donee in a lower marginal income tax bracket. The donor retains security in the gifted property. The lease payments made by the donor to the donee are considered additional gifts.

Statement II is false because in a gift-leaseback the donor relinquishes security and control of the gifted property. Statement IV is false because lease payments are income to the donee, not additional gifts.

Which of the following statements correctly identify estate planning activities that can be performed by a financial planner who is not also a licensed attorney? Advise a client as to who would receive property under the state intestacy statutes Estimate a client's potential federal gift tax liability Advise a client that he or she needs a new will Draft a living will for a client to execute

Statements I and IV are actions only a licensed attorney can perform. Planners can certainly estimate gift tax liability and determine if a current will meets client estate planning goals.

Reed, age 45, has come to you for help in planning his retirement. He works for a manufacturing company, where he earns a salary of $75,000. Reed would like to retire at age 65. He feels this is a realistic goal because he has consistently earned 9% on his investments and inflation has only averaged 3%. Assuming he is expected to live until age 90 and he has a wage replacement ratio of 80%, how much will Reed need to have accumulated on the day that he retires to adequately provide for his retirement lifestyle?

The answer is $1,490,649. Step 1: Determine the present value of capital needs: Current income$75,000Wage replacement ratio× 80%Present value of capital needs$60,000 Step 2: Determine the future value of the capital needs in the first year of retirement: PV of capital needs($60,000)n (number of years until retirement)20i (use inflation rate)3FV (required income in the first year of retirement)$108,366.6741 Step 3: Determine the amount of savings (capital) needed at retirement to fund expenses throughout remainder of life expectancy: PMTAD(annuity due)($108,366.6741)n (retirement life expectancy)25 (90 − 65)i (use real rate of return)5.8252 [(1.09 ÷ 1.03) − 1] × 100PV (capital needed at retirement)$1,490,649 LO 8.3.2

Reed, age 45, has come to you for help in planning his retirement. He works for a manufacturing company, where he earns a salary of $75,000. Reed would like to retire at age 65. He feels this is a realistic goal because he has consistently earned 9% on his investments and inflation has averaged 3%. If Reed expects to live until age 90 and he has a wage replacement ratio of 80%, assuming a capital preservation approach, how much will Reed need to have accumulated on the day that he retires to adequately provide for his retirement lifestyle?

The answer is $1,663,516. Step 1: Determine the present value of capital needs: Current income$75,000Wage replacement ratio× 80%Present value of capital needs$60,000 Step 2: Determine the future value of the capital needs in the first year of retirement: PV of capital needs($60,000)n (number of years until retirement)20i (use inflation rate)3%FV (required income in the first year of retirement)$108,366.6741 Step 3: Determine the amount of savings (capital) needed at retirement to fund expenses throughout remainder of life expectancy using a capital utilization approach: PMTAD(annuity due) ($108,366.6741) n (retirement life expectancy)25 (90 − 65)i (use real rate of return)5.8252 [(1.09 ÷ 1.03) − 1] × 100PV (capital needed at retirement)$1,490,649 Step 4: For capital preservation, calculate the additional capital needed at retirement to generate the desired retirement income and leave a balance at life expectancy equal to the original capital utilization value: FV = $1,490,649 n = 25 i = 9 Solve PV = (172,867) Add to previous step: $1,490,649 + $172,867 = $1,663,516 LO 8.3.2

Thad and Debra, both age 48, are married and will file a joint return. Their 2020 modified adjusted gross income is $120,000, (including Thad's $95,000 salary). Debra had no earned income of her own. Neither spouse was covered by an employer-sponsored retirement plan. What is the total maximum deductible contribution Thad and Debra may make to a traditional IRA this year?

The answer is $12,000. Because neither Thad nor Debra participates in an employer-sponsored retirement plan, they can contribute and deduct $6,000 each for 2020. While Debra has no earned income, a spousal IRA may be established and funded based on Thad's compensation.

Sally, age 37, works for two employers, ABC Corporation and XYZ Corporation, both of which maintain Section 401(k) plans. If Sally defers $6,000 to ABC's Section 401(k) plan in 2020, how much can she then defer to XYZ's plan this year?

The answer is $13,500. The maximum allowable elective deferral for 2020 is $19,500. If Sally contributes $6,000 to ABC's plan, then she can only contribute up to $13,500 to XYZ's plan ($19,500 − $6,000 = $13,500).

John Irving, the 55-year old owner of ABC Corporation, wants to implement a new comparability plan. John's salary is $150,000. The remaining eligible participant census is as follows: AgeSalaryEmployee A35$50,000Employee B33$45,000Employee C54$60,000Employee D41$35,000Employee E43$36,000 If John wants to contribute an aggregate total of $41,300 to the plan this year, what is the maximum amount John can contribute to the comparability plan for himself?

The answer is $30,000. A new comparability plan will only satisfy the nondiscrimination rules if the plan design satisfies one of either of these: Each eligible non-highly compensated employee (HCE) must receive an allocation of at least 5% of compensation. If the plan provides for an allocation rate of less than 5%, the minimum allocation rate for the non-HCEs is one-third of the highest allocation rate under the plan. John can contribute a maximum $30,000 (20%) of his salary to the plan while limiting his other employees to as little as 5% of salary. In this example, the total compensation of the eligible employees is $226,000. A 5% contribution for this group totals $11,300, leaving $30,000 of the total $41,300 for John's benefit. LO 3.2.1

Steve retired from ABC Corporation this year and received a lump-sum distribution from ABC's qualified retirement plan. The distribution consisted entirely of ABC stock valued at $200,000 on the date of distribution. The fair market value of the stock at the time of contribution to the plan was $80,000. Assuming Steve does not sell the stock this year, what amount is included in Steve's gross income this year as a result of the distribution?

The answer is $80,000. Because the distribution is a lump-sum distribution of employer stock, the net unrealized appreciation (NUA) concept applies. Under the NUA rules, the adjusted basis of the stock to the plan trust ($80,000) is included in Steve's gross income in the year of the distribution and is treated as ordinary income. When Steve later sells the stock, he will have an $80,000 basis in it.

Roderick Manufacturing maintains a qualified defined benefit pension plan. There are 100 eligible employees working for the company. What is the minimum number of employees the retirement plan must cover to satisfy the 50/40 test?

The answer is 40. Under the 50/40 test, a defined benefit plan must cover the lesser of 50 employees or 40% of all eligible employees. In this case, the lesser of 50 employees or 40% of all eligible employees (100) is 40 employees. One way to remember the 50/40 test is the phrase "people before percentages" (50 people or 40%). Also, note that there are no qualifiers to the types of people. It is not 50 non-highly compensated people. It is just 50 employees who work for Roderick Manufacturing.

Ross Company has a traditional Section 401(k) plan. The actual deferral percentage (ADP) for all eligible non-highly compensated employees (non-HCEs) is 4%. What is the maximum ADP for the highly compensated employees (HCEs) group at Ross Company?

The answer is 6%. The maximum ADP for HCEs at the Ross Company is 6%. To satisfy the ADP test, a traditional 401(k) plan must meet one of the following two tests. The ADP for eligible HCEs must not be more than the ADP of all other eligible employees multiplied by 1.25. In this case, the non-HCEs averaged 4%. (4% × 1.25 = 5%) The ADP for eligible HCEs must not exceed the ADP for other eligible employees by more than 2% (4% + 2% = 6%), and the ADP for eligible HCEs must not be more than the ADP of all other eligible employees multiplied by 2 (4% × 2 = 8%). This second test is a lesser-than test. The lesser number is 6%. Thus, 6% is the second test answer. Thus, the two test answers are 5% and 6%. The plan only must pass one of these tests, and the HCEs want to know how much they can contribute, so the higher number always wins. That is 6% in this case. LO 3.3.2

Martha has been impressed with the appreciation of the coin collection she received as a gift from her mother and would like to take advantage of this by using coins as an investment in the IRAs. Which of the following statements regarding coins as investments in IRAs is CORRECT?

The answer is American Eagle gold coins are permitted IRA assets. Only permissible collectible that an IRA may invest in is certain U.S. coins, such as the American Eagle gold coin.

Required minimum distributions from a traditional IRA must begin no later than

The answer is April 1 of the year following the year in which the IRA owner attains age 70½. The Internal Revenue Code provides that minimum distributions from a traditional IRA must begin no later than April 1 of the year following the year in which the IRA owner attains age 70½.

While Section 403(b) (tax-sheltered annuity plan or TSA) plans are an excellent source of retirement savings, they do have some disadvantages, such as investments are limited to mutual funds and annuities. Section 403(b) plans must comply with the actual contribution percentage (ACP) test for employer matching contributions. actual deferral percentage (ADP) testing causes Section 403(b)/TSA plans to be relatively costly and complex to administer. account balances at retirement age are guaranteed to be sufficient to provide adequate retirement amounts for employees who entered the plan at later ages.

The answer is I and II. Section 403(b)/TSA plan investments are limited to mutual funds and annuities. Although the ADP test does not apply, Section 403(b)/TSA plans must comply with the ACP test for matching contributions. One way to remember that Section 403(b) plans must pass ACP testing and not ADP testing is that Section 403(b) plans are for 501(c)(3) organizations. The (c) in 501(c)(3) is like the "C" in ACP testing. Nondiscrimination testing causes Section 403(b)/TSA plans to be relatively costly and complex to administer. Account balances at retirement age may not be sufficient to provide adequate retirement amounts for employees who entered the plan at later ages.

Jack inherited his father's Section 401(k) plan account. Which of the following statements regarding Jack's options is (are) CORRECT? Jack is permitted to use a direct trustee-to-trustee transfer of the plan balance into an inherited IRA account. If Jack creates an inherited IRA with the benefit, he can designate his own beneficiary to the account. The payout of the benefit in an inherited IRA is over the lifetime of the original account holder, Jack's father.

The answer is I and II. Statement III is incorrect. As a non-spouse beneficiary, Jack is required to receive distributions over his own remaining life expectancy, reduced by 1 each year.

Which of the following statements regarding integrating a plan with Social Security are NOT correct? Only the excess method can be used by a defined benefit pension plan. The maximum increase in benefits for earnings above the covered compensation level is 26.25% for a defined benefit pension plan. Because there is a disparity in the Social Security system, all retirement plans are allowed integration with Social Security. Under the offset method, a fixed or formula amount approximates the existence of Social Security benefits and reduces the plan formula.

The answer is I and III. Statement I is incorrect because the excess method may only be used by a defined contribution plan. A defined benefit pension plan may use either the excess method or the offset method for Social Security integration. Statement III is incorrect because not all retirement plans may be integrated with Social Security.

Blake, age 72, is required to take substantial required minimum distributions (RMDs) from his qualified retirement plan. He has no current need for the income and wants to decrease the amount of the distributions without incurring a penalty. Blake is not interested in a lump-sum distribution from the plan at this time. Which of the following statements regarding Blake's options is CORRECT? Blake may take a distribution in addition to his RMD from his qualified plan and convert the additional distribution to a Roth IRA within 60 days. Blake cannot roll over retirement plan proceeds to a traditional IRA after age 70½.

The answer is I only. If Blake takes a distribution that is in addition to his RMD, he can pay the required income tax on this distribution and convert it to a Roth IRA. Because there are no required minimum distributions for Roth IRAs, Blake will have effectively reduced the amount of his pretax retirement plan account against which he must calculate subsequent RMD. Statement II is incorrect. A direct transfer or rollover may occur after age 70½ but no new contributions can be made after age 70½.

Pension Benefit Guaranty Corporation (PBGC) insurance coverage is required for which of the following plans? Traditional defined benefit pension plan Target benefit pension plan Money purchase pension plan Profit-sharing plan

The answer is I only. Money purchase pension plans, profit-sharing plans, and target benefit pension plans do not require PBGC insurance because they are forms of defined contribution plans. Only defined benefit pension plans (traditional defined benefit plans and cash balance plans) require the payment of PBGC insurance premiums.

Which of the following statements describing how qualified pension plans differ from SEP and SIMPLE plans is(are) CORRECT? Qualified plan rules provide greater flexibility in the number and makeup of the employees covered by the plan than do the rules pertaining to SEP and SIMPLE plans. Participants must be fully and immediately vested in the contributions to qualified plans, but SEP and SIMPLE plans are permitted to have vesting schedules.

The answer is I only. Statement II is incorrect because participants must be fully and immediately vested in the contributions to SEPs and SIMPLEs. Qualified plans can include vesting schedules.

Which of the following statements regarding the tax effects of converting a traditional IRA to a Roth IRA is (are) CORRECT? The converted amount is treated as a taxable distribution from the traditional IRA. The 10% premature penalty applies if the owner is not at least 59½ years old.

The answer is I only. When a traditional IRA is converted to a Roth IRA, the converted amount is treated as a taxable distribution and is included in the owner's gross income. The 10% penalty does not apply to the conversion amount when converted, regardless of the owner's age. However, if the taxable portion of the converted amount is withdrawn within five years of the conversion, then the taxable portion of the conversion is treated as coming out first when the converted amount is withdrawn. This taxable amount would be subject to the early withdrawal rules and penalized 10% unless an exception applies. The point of this rule is to protect Roth conversions from being sham transactions intended to get around the 10% early withdrawal penalty. The law treats withdrawals of converted amounts that are more than five years past the conversion date the same as contributions.

Which of the following are considered profit-sharing plans? Stock bonus plan Traditional Section 401(k) plan New comparability plan Employee stock option plan (ESOP)

The answer is I, II, III, and IV. All of these are considered profit-sharing plans. They don't require fixed annual employer contributions and are not affected by the minimum funding standard requirements.

Which of the following are true of the actual contribution percentage test (ACP) test for 401(k) plans? The ACP test is not used unless a 401(k) plan has a match or allows employee after-tax contributions. The ACP test uses the same two test structure and percentage rules as the ADP test. The ADP test accounts only for employee deferrals. The ACP test accounts for employer matching and after-tax contributions, but not pretax contributions and elective deferrals. If the ADP of the non-highly compensated employees is greater than 2% but less than or equal to 8%, then the maximum ADP of the highly compensated employees is 2% more than the ADP of the non-highly compensated employees.

The answer is I, II, III, and IV. All statements are true.

A business owner-client approaches a financial planner for advice on selecting a retirement plan for the business. What factors should guide the financial planner's recommendations? The owner's retirement savings need The owner's current age The amount of risk the client is comfortable assuming The financial stability of the business.

The answer is I, II, III, and IV. All the factors listed should be considered in selecting a retirement plan for the business.

Which of the following retirement plans can be integrated with Social Security? Profit-sharing plan Simplified employee pension (SEP) plan Money purchase pension plan Defined benefit pension plan

The answer is I, II, III, and IV. All these plans may be integrated with Social Security. Employee stock ownership plans (ESOPs), savings incentive match plan for employees (SIMPLEs), and salary reduction SEPs (SARSEPs) are not permitted to use integration. Also, employee elective deferrals and employer matching contributions cannot be integrated.

Section 401(k) plans must have automatic survivor benefits (QJSAs and QPSAs) unless the plan provides that, upon the participant's death, the vested account balance will be paid in full to the surviving spouse. the plan is not a direct or indirect transferee of a plan to which the automatic survivor annuity requirements apply. the participant elects to receive payment as a lump-sum distribution. the participant does not elect payments in the form of a life annuity.

The answer is I, II, III, and IV. For a participant to have elected to receive a lump-sum distribution, the spouse must sign a spousal consent form in front of a notary or an authorized plan representative.

Which of the following may be eligible for rollover treatment? A total distribution from a Section 401(k) plan A distribution from an IRA The nontaxable portion of qualified plan distribution A required minimum distribution payment

The answer is I, II, and III. All IRA and qualified plan distributions are eligible for rollover treatment except distributions made to satisfy the minimum distribution rules and distributions made as part of a series of substantially equal periodic payments. Required minimum distributions are not eligible for rollover treatment. The law states a nontaxable portion of a qualified plan distribution must be made using a trustee-to-trustee transfer because that is the only reliable way to provide documentation that this money was actually nontaxable.

A simplified employee pension (SEP) plan requires employer contributions on a nondiscriminatory basis. can be integrated with Social Security. cannot deny participation to any employee 21 years of age or older based on age. imposes mandatory employer contributions.

The answer is I, II, and III. Only statement IV is incorrect. A SEP plan is a retirement plan that uses an IRA as the receptacle for employer/employee contributions. The SEP plan is often a good choice for very small companies because of its low cost and ease of administration. All employer contributions to a SEP plan are discretionary.

Which of the following are examples of qualified retirement plans? Cash balance plan Section 401(k) plan Section 403(b) plan Employee stock ownership plan (ESOP)

The answer is I, II, and IV. A Section 403(b) plan is a tax-advantaged plan but not an ERISA-qualified retirement plan. While tax-advantaged plans are very similar to qualified plans, there are some minor differences. For example, a tax-advantaged plan is not allowed to have net unrealized appreciation (NUA) treatment. They are also not allowed to offer 10-year forward averaging or special pre-1974 capital gains treatment. Tax-advantaged plans also have less restrictive nondiscrimination rules. Otherwise they are very similar to qualified plans.

Henry works for an accounting firm that sponsors a Section 401(k) plan. Henry, who has a current salary of $35,000, was hesitant to contribute to the plan because in the past he felt as though he may need the money before retirement. At a recent employer-sponsored seminar, Henry learned that he could receive a loan from his Section 401(k) plan without paying any income tax. Henry is now considering making pre-tax elective deferrals to the Section 401(k) plan, but he wants to know more specific details regarding loan provisions. Which of the following statements regarding nonpenalized loans from qualified plans is (are) CORRECT? The limit on loans is generally one-half of the participant's vested account balance not to exceed $50,000. The limit on the term of any loan is generally five years. If an employee leaves the company, a retirement plan loan may be rolled over to an IRA and the participant may continue making the loan payments as planned. Loans to a 100% owner-employee are permissible.

The answer is I, II, and IV. Statement I is correct. Generally, loans are limited to one-half the vested account balance and cannot exceed $50,000. Note: When account balances are less than $20,000, however, loans up to $10,000 are available. Statement II is correct. The limit on the term of any loan is generally five years, unless the loan is for a principal residence. Loans for the purpose of buying a residence must be repaid over a reasonable period. Statement III is incorrect. A qualified plan loan may not be rolled over to an IRA. Any outstanding loan balance is treated as a distribution and thus is subject to income taxes and the early withdrawal penalty rules. Statement IV is correct. Loans from qualified plans to sole proprietors, partners, shareholders in S corporations and C corporations are permitted.

Which of the following statements regarding fully insured Section 412(e)(3) plans is(are) CORRECT? A fully insured plan is inappropriate for an employer who cannot commit to regular premium payments. This type of plan is not required to be certified by an enrolled or licensed actuary. All Section 412(e)(3) plans must meet minimum funding standards each plan year. A Section 412(e)(3) plan is a type of defined benefit pension plan.

The answer is I, II, and IV. Statement III is incorrect. Section 412(e)(3) plans must only meet minimum funding standards if there is a loan outstanding against the insurance policy funding the plan.

Build Corporation communicated several goals to its financial adviser when it decided to implement a qualified retirement plan for the company. After reviewing these goals, the adviser recommended that Build Corporation implement a defined benefit pension plan. Which of the following statements regarding employers who are candidates for a defined benefit pension plan are CORRECT? Typically, they have the objective of instituting a plan with highly predictable costs. They indicate that the desire to provide a tax shelter for key employees outweighs the need for an administratively convenient plan. They want benefit levels guaranteed. They want a simple and inexpensive plan.

The answer is II and III. Employers who sponsor defined benefit pension plans typically do not have highly predictable costs, because funding is subject to an annual actuarial determination. Highly paid employees receive a tax shelter with a defined benefit plan in the sense that their compensation includes large contributions to the defined benefit and they are not subject to current income tax on this compensation. The Pension Benefit Guaranty Corporation (PBGC) and the employer guarantee benefit levels. Defined benefit pension plans are complex to design and expensive to install and maintain.

Which of the following are minimum coverage tests for qualified retirement plans? Nondiscrimination test Average benefits percentage test Ratio test Maximum compensation test

The answer is II and III. The two minimum coverage tests for qualified retirement plans are the average benefits percentage test and the ratio test. In order to be qualified, a retirement plan must meet at least one of these tests if the plan does not meet the percentage (safe harbor) test.

Basic provisions of SIMPLE IRAs include which of the following? They are subject to actual deferral percentage test (ADP) nondiscrimination rules. Employees are 100% vested in their elective deferrals. Employees are not fully vested in employer contributions until completing five years of service. Employers with fewer than 100 employees who earned $5,000 during any two preceding years and are reasonably expected to earn at least $5,000 during the current year must be allowed to participate.

The answer is II and IV. Employees are 100% vested in their elective deferrals. Employers with fewer than 100 employees who earned $5,000 during any two preceding years and are reasonably expected to earn at least $5,000 during the current year must be allowed to participate. SIMPLE IRAs are not subject to the nondiscrimination rules generally applicable to qualified plans (including top-heavy rules). The employee is 100% vested in both his elective deferrals as well as any employer contributions.

Which of the following statements regarding Section 457 plans is (are) CORRECT? Deductibility of plan contributions is an important factor for employers choosing a Section 457 plan to consider. Earnings on assets in a Section 457 plan grow tax-deferred until withdrawn. Required minimum distribution rules do not apply. A Section 457 plan is a nonqualified deferred compensation plan.

The answer is II and IV. Statement I is incorrect. Deductibility of plan contributions is not a factor for employers choosing a Section 457 plan because these employers are tax-exempt. Statement II is correct. Earnings on assets in a Section 457 plan grow tax-deferred until withdrawn. Statement III is incorrect. Required minimum distribution rules apply. Statement IV is correct. A Section 457 plan is a nonqualified deferred compensation plan.

Which of the following persons could make tax-deductible contributions to a traditional IRA regardless of their modified adjusted gross income (MAGI)? A person who participates in a SEP IRA A person who participates in a Section 457 plan A person who participates in a Section 401(k) plan A person who participates in a Section 403(b) plan

The answer is II only. A person who participates in a qualified plan, SEP IRA, or Section 403(b) plan may not be able to make tax-deductible IRA contributions if the participant's MAGI exceeds certain limits. Participation in a Section 457 plan does not subject a person to these limitations.

Which of the following statements regarding the coverage rules for qualified plans is (are) CORRECT? The coverage tests for qualified plans include the percentage test, the ratio test, and the average contribution percentage test. A retirement plan can cover any portion of the workforce, provided it satisfies one of the three coverage tests under Section 410(b).

The answer is II only. Statement I should list the third coverage test as the average benefit percentage test, not the average contribution percentage test.

Sally, age 60, has received a $50,000 distribution from $100,000 she is to receive from her ex-husband's (who is age 55) qualified plan account under a qualified domestic relation order (QDRO). Which of the following statements is CORRECT regarding the QDRO and the distributed funds? Irrespective of the plan document, Sally may demand an immediate cash distribution of the remaining funds from the plan trustees. She may roll over the $50,000 distribution into an IRA. Sally's ex-husband is not subject to an early distribution penalty in the execution of the QDRO. Sally may be required to leave the remaining funds with the plan trustee until the earliest time for distributions under the plan.

The answer is II, III, and IV. A trustee may not be forced to distribute assets from a plan unless the plan document allows for it.

When she retired at age 64, Lauren received a lump-sum distribution from her employer's stock bonus plan. The fair market value of the employer stock contributed to her account was $200,000 at the time of contribution. At the time of the distribution, the employer stock in Lauren's account had a fair market value of $300,000. Six months later, Lauren sold the stock for $310,000. Which of the following statements regarding the sale of Lauren's stock is(are) CORRECT? The $300,000 distribution is taxed at the long-term capital gain rate. Lauren has a $10,000 short-term capital gain when the stock is sold. There was no income tax liability incurred when the stock was contributed to the plan. The net unrealized appreciation (NUA) on the stock is $100,000.

The answer is II, III, and IV. Of the $300,000 Lauren received as a lump-sum distribution from the stock bonus plan, $100,000 is net unrealized appreciation (NUA) and will be taxed at the long-term capital gain rate. The remaining $200,000 is taxed at Lauren's ordinary income tax rate in the year of the lump-sum distribution. Because Lauren sold the stock within six months of distribution, the $10,000 post-distribution appreciation is taxed as short-term capital gain.

Adam, age 48, and Mary, age 47, were married for 15 years when they divorced last year. Adam died this year. They have two young children, ages 10 and 12, who are cared for by Mary. Adam's 70-year-old mother, Sarah, also survived him. At the time of Adam's death, he was currently, but not fully, insured under Social Security. What benefits are Adam's survivors entitled to under the Social Security program? A dependent parent's benefit A lump-sum death benefit of $255 A children's benefit based on a percentage of Adam's primary insurance amount (PIA) A surviving spouse benefit to take care of a dependent child

The answer is II, III, and IV. One lump-sum death benefit of $255 is payable if he was fully or currently insured. The children's benefit is payable because Adam was currently insured. Adam's divorced spouse (the children's mother) is entitled to a caretaker's benefit for caring for his children under age 16. Adam's mother would only be entitled to a benefit if Adam was fully insured and he had been providing at least half of her support at the time of his death.

Which of the following retirement plans generally permit in-service withdrawals at any age? Money purchase pension plan Profit-sharing plan Section 401(k) plan SEP plan

The answer is II, III, and IV. Pension plans (traditional defined benefit, cash balance, target benefit, or money purchase) generally do not allow in-service withdrawals to participants under the age of 62. All the others listed allow for in-service withdrawals if the plan document permits.

Which of the following statements regarding prohibited transactions is CORRECT? The lending of money or other extension of credit between the plan and a party in interest (outside of nondiscriminatory retirement plan loans based on the participant's account balance) is not a prohibited transaction. One category of prohibited transactions involves self-dealing. One category of prohibited transactions bars a fiduciary from causing the plan to engage in a transaction if the fiduciary knows or should know that such a transaction constitutes a direct or indirect involvement between the plan and the parties in interest. One category of prohibited transactions involves the investment in the sponsoring employer's stock or real property above certain limits.

The answer is II, III, and IV. Statement I is incorrect. The sale, exchange, lending, or leasing of any property between the plan's assets and a party in interest is a prohibited transaction. A plan participant can take a retirement plan loan based on the participant's balance, but the plan cannot make a loan of the general plan assets to a party in interest.

If a covered worker were to become disabled for Social Security benefit purposes, which of the following individuals would be eligible to also receive a benefit from Social Security based on the disabled worker's record? A dependent parent of the disabled worker, age 62 or over The disabled worker, age 62 or over The spouse of the disabled worker An unmarried dependent child of the disabled worker, under age 19 and still in high school

The answer is II, III, and IV. The dependent parent of a disabled worker is not eligible for benefits under the disabled worker's employment record when the worker receives Social Security disability benefits.

Several years ago, Greener Grass Company implemented a traditional defined benefit plan. According to the plan document, the employer must contribute an annual amount that will provide the employees with a specified benefit at retirement. Which of the following events would be expected to decrease the employer's annual contribution to a traditional defined benefit pension plan using a percentage for each year of service benefit formula? Inflation is higher than expected. Benefits are cost of living adjusted. Forfeitures are higher than anticipated. The investment returns of the plan are greater than expected.

The answer is III and IV. Defined benefit pension plan contributions decrease because of the events described in statements III and IV. Statements I and II are incorrect. Inflation would likely cause salaries and plan expenses to increase, thereby causing contributions to increase. Likewise, benefits that are adjusted for the cost of living would result in greater employer contributions, not less.

Todd, age 60, has made contributions of $75,000 to his traditional IRA, of which $15,000 were nondeductible contributions. He is considering taking a $20,000 distribution from his IRA, which currently has a fair market value of $175,000. When calculating the nontaxable portion of his IRA, which of the following formulas is CORRECT? Nontaxable portion = nondeductible contributions ÷ [(IRA balance at the beginning of the year + the IRA balance at the end of the year) ÷ 2] × IRA distributions Nontaxable portion = nondeductible contributions ÷ (IRA balance at the beginning of the year + any distributions taken during the year) × IRA distributions Nontaxable portion = [(nondeductible contributions prior to current year + all contributions for current year) ÷ (balances at end of current year + distributions received in current year)] × total distributions during current year

The answer is III is only. The correct formula is: nontaxable portion = [(nondeductible contributions prior to current year + all contributions for current year) ÷ (balances at end of current year + distributions received in current year)] × total distributions during current year LO 5.3.1

Bill's employer maintains a target benefit pension plan. Bill is age 59. The plan was originally designed to benefit a 38-year-old key employee. There is also substantial turnover at Bill's company. Which of the following statements is (are) CORRECT? Bill knows exactly what retirement benefit to expect. Bill's retirement benefit is funded through elective deferrals. Forfeitures are likely to be allocated equally to Bill and the 38-year-old employee. Contributions to the plan are mandatory.

The answer is IV only. Benefits depend on such plan's account balances, and the final benefit amount is not guaranteed. Target benefit pension plans are funded by the employer, not through employee elective deferrals. Forfeitures in such plans are likely to be unequal as a result of unequal compensation.

Which of the following statements regarding profit-sharing plans is (are) CORRECT? Profit-sharing plans are only suited for companies with predictable cash flows. Company profits are required to make contributions to a profit-sharing plan. Companies adopting a profit-sharing plan are required to make annual contributions to the plan. The maximum tax-deductible employer contribution to a profit-sharing plan is 25% of total covered employee compensation.

The answer is IV only. Statements I, II, and III are incorrect. Profits are not required to be able to make a contribution to a profit-sharing plan. A current contribution may be made from retained earnings or current cash flow. Annual contributions are not required in a profit-sharing plan. Profit-sharing plans are suitable for companies with unstable earnings given that they have discretion over contributions.

For tax-exempt employers who do not want to implement a Section 457 plan and desire a plan funded strictly by employee elective deferrals, a good alternative would be

The answer is Section 403(b) plan. A Section 403(b) plan, like the Section 457 plan, can be used as an employee deferred-contribution plan. Certain tax-exempt employers can implement Section 403(b) plans. With a SEP plan or a profit-sharing plan, there are also employer contribution considerations. New SARSEP plans can no longer be established.

Armor Company has implemented an age-based profit-sharing plan. Under this plan,

The answer is allocations to participants are made in proportion to the participant's age-adjusted compensation. A participant's compensation is age-adjusted by multiplying the participant's actual compensation by a discount factor based on the participant's age and the interest rate elected by the plan sponsor. As a result, older employees generally receive the greatest allocation under an age-based profit-sharing plan. Nondiscrimination rules are tested in accordance with benefits rather than contributions. The final retirement benefit is not guaranteed in any type of profit-sharing plan.

All the following statements describe situations in which a target benefit pension plan would best suit the company except

The answer is an employee census showing young owners and young rank-and-file employees. A good candidate for a target benefit pension plan is a business that has an employee census showing older owners who are around 50 or older and younger rank-and-file employees. The other choices are additional characteristics that describe good candidates for target benefit pension plans.

Why would a qualified retirement plan include real estate among its portfolio of investment assets?

The answer is as an inflation hedge. The inclusion of real estate among a retirement plan's investment portfolio is most suitable as a hedge against future inflation. Otherwise, it is generally nonliquid, is not stable enough to fund fixed obligations, and is not always considered a very low risk holding.

Which of the following statements regarding a stretch IRA is CORRECT? It allows the IRA owner's beneficiary to name his own beneficiary upon the owner's death. It extends or stretches the period of tax-deferred earnings within an IRA possibly over several decades.

The answer is both I and II. A stretch IRA extends or stretches the period of tax-deferred earnings within an IRA beyond the lifetime of the original owner, possibly over several decades.

A traditional IRA is appropriate when it is considered an important supplement or alternative to a qualified pension or profit-sharing plan. sheltering current compensation or earned income from taxation is a taxpayer's goal.

The answer is both I and II. A traditional IRA is also suitable when a taxpayer wishes to accumulate assets for retirement or defer taxes on investment income.

Which of the following statements regarding plan forfeitures in a money purchase pension plan is(are) CORRECT? Plan forfeitures may be used to reduce future employer contributions. An employer may reallocate the plan forfeitures among the remaining plan participants, increasing their potential individual account balances but only up to the annual additions limit for each participant.

The answer is both I and II. Both of these statements are correct for money purchase pension plans.

Which of the following regarding Social Security funding is (are) CORRECT? Social Security is funded through a series of taxes paid by the participant and the participant's employer. Social Security is funded through a Federal Insurance Contributions Act (FICA) tax and the self-employment (SECA) tax.

The answer is both I and II. Social Security is funded through a series of taxes paid by both the participant and participant's employer, commonly called payroll taxes. A self-employed individual must pay both the employer and employee portions of the FICA tax, known as the self-employment (SECA) tax.

A fully insured Section 412(e)(3) pension plan is funded exclusively by

The answer is cash value life insurance or annuity contracts. A fully insured 412(e)(3) pension plan is funded exclusively by cash value life insurance or annuity contracts. Using insurance as a funding vehicle ensures the payment of a death benefit to plan beneficiaries.

A preretirement distribution from a qualified retirement plan can escape the 10% early withdrawal penalty in each of the following situations except

The answer is distributions made after a separation from service for early retirement after any age. For a preretirement distribution to escape the 10% penalty for early distribution, the distribution must be made after a separation from service in the year the worker turns age 55 or later. This exception is not applicable to IRAs.

Napoleon Enterprises sponsors a SIMPLE 401(k) for its employees. Under the plan, the company matches employee contributions up to 3% of compensation. Which of the following statements about Napoleon Enterprises' SIMPLE 401(k) is CORRECT?

The answer is employees cannot make after-tax contributions to the plan. Employee after-tax contributions are not allowed. All the other statements are incorrect: Unlike SIMPLE IRAs, employers that sponsor SIMPLE 401(k)s cannot reduce the matching percentage to below 3%. Employer contributions to a SIMPLE 401(k) are 100% vested. The 25% penalty applies only to SIMPLE IRAs.

To qualify for disability income benefits under Social Security, a worker must have an impairment that

The answer is expected to last at least 12 months or result in death. To qualify for Social Security disability income benefits, a person must suffer an impairment that is expected to last at least 12 months or result in death. The disability also must have lasted at least 5 months before Social Security disability benefits can be paid.

Which of the following is one of the differences between defined benefit pension plans and defined contribution plans?

The answer is investment risk is borne by the employer in a defined benefit pension plan, whereas the employee bears the risk in a defined contribution plan. A defined benefit pension plan has a benefit limit, whereas a defined contribution plan has a contribution limit. Investment risk is borne by the employer in a defined benefit pension plan, whereas the employee bears the risk in a defined contribution plan. Accounts are commingled in a defined benefit pension plan, and a defined contribution plan maintains separate accounts for each participant. A guaranteed retirement benefit is the goal of a defined benefit pension plan, while a guaranteed contribution is the focus of a defined contribution plan.

Which of the following statements regarding Roth IRAs and pre-tax 401(k) plans is (are) CORRECT? Roth IRAs require distributions no later than age 70 ½ while the participant is living. There is not an income limitation to participate in a pre-tax 401(k) plan or Roth IRA.

The answer is neither I nor II. Although there is not an income limitation to participate in a pre-tax 401(k) plan, there are income limits for Roth IRAs (2020 - modified AGI: $206,000 married/$139,000 single). For Roth IRAs, there is no requirement to start taking distributions while the participant is living. For pre-tax 401(k) plans, distributions must begin no later than age 70 ½, unless the participant is still working and not a 5% owner.

Regarding assumptions used in retirement needs analysis calculations, which of the following is (are) CORRECT? All other things being equal, increasing the life expectancy of the retiree will lower the amount of capital needed on the first day of retirement to support the assumed retirement income. All other things being equal, changing the assumed rate of return from 6% to 8% and the assumed inflation rate from 2% to 4%, will lower the amount of capital needed on the first day of retirement to support the assumed retirement income.

The answer is neither I nor II. Increasing the life expectancy of the retiree increases the amount of capital needed on the first day of retirement to support the assumed retirement income because the retiree will draw on the capital fund for a longer period. Statement II is incorrect because an 8% investment return and 4% inflation rate produces a lower inflation-adjusted rate of return, and thus increases the amount of capital needed on the first day of retirement to support the assumed retirement income. [(1.06 ÷ 1.02) − 1] × 100 = 3.9216%; [(1.08 ÷ 1.04) − 1] × 100 = 3.8462% LO 8.3.1

Robert established a Roth IRA. He turns age 70½ this year. Which of the following statements is (are) CORRECT? Robert must begin taking required minimum distributions (RMDs) by April 1 of next year. Robert can no longer make contributions to the Roth IRA.

The answer is neither I nor II. The original owner of a Roth IRA is never subject to an RMD requirement during his lifetime and can continue making contributions after he reaches age 70½.

The Department of Labor (DOL) issues

The answer is rulings including prohibited transaction exemptions (PTEs). The Department of Labor issues advisory opinions and rulings (including prohibited transaction exemptions) similar to private-letter rulings, which are issued by the IRS. The DOL does not issue guaranty insurance. The summary plan description is required by the DOL, but the DOL does not approve plan documents. The IRS approves plan documents.

ABC Company would like to establish a retirement plan incorporating the following objectives: Attract and retain employees. The employer will make all contributions to the plan with company stock. The plan must integrate with Social Security. What type of retirement plan best suits ABC's objectives?

The answer is stock bonus plan. A stock bonus plan would accomplish ABC Company's objectives. The others are not the best plan for these reasons: Money purchase pension plans only allow for 10% company stock. ESOPs cannot be integrated with Social Security. New SARSEPs can no longer be established.

A state or local government would choose to establish a Section 457 plan for all the following reasons except

The answer is tax deductibility of employer contributions. Because Section 457 plans are sponsored by tax-exempt entities, deductibility of plan contributions is not an issue and would not be a reason to establish such a plan. A Section 457 plan is not a qualified plan and has no early withdrawal penalty on distributions.

Which of the following statements is NOT correct regarding the conversion of a traditional IRA to a Roth IRA?

The answer is the IRA owner's modified adjusted gross income (MAGI) cannot exceed $100,000 in the year of the conversion. There is no MAGI limit for a taxpayer in the year in which there is a conversion.

Under a profit-sharing plan,

The answer is the company has flexibility as to annual funding. Pension plans can invest up to 10% only of plan assets in employer stock. Profit-sharing plans have no restrictions regarding investment in employer stock. The employer may deduct a contribution limited to only 25% of participating employees' covered compensation. The employer must make substantial and recurring employer contributions, or the IRS will remove the plan's qualified status. The employee bears investment risk.

Total annual contributions to an individual participant in a traditional Section 401(k) plan are limited in 2020 to

The answer is the lesser of 100% of compensation, or $57,000. Total annual contributions to a participant's account are limited to the lesser of 100% of employee compensation, or $57,000 (2020 with only the first $285,000 (2020) of employee compensation considered in the contribution formula. The total contribution is made up of the worker contribution, the employer contribution and reallocated forfeitures. The worker contribution alone is limited to $19,500 in 2020 for those 49 and younger.

Because a simplified employee pension (SEP) plan is not a qualified plan, it is not subject to all the same rules as qualified plans; however, it is subject to many of the same rules. Which of the following statements when comparing or contrasting a SEP plan to a qualified plan is CORRECT?

The answer is the maximum contribution possible on behalf of an individual participant is $57,000 (2020). Both SEP plans and qualified plans can be funded as late as the due date of the return plus extensions. The maximum contribution possible on behalf of an individual participant is $57,000 (2020). Qualified plans are protected under ERISA and federal bankruptcy law. SEPs share this protection. Both types of plans have the same nondiscriminatory and top-heavy rules.

Jill has decided to offer a retirement plan to her employees. She wants to implement a savings incentive match plan for employees (SIMPLE) and is trying to decide between a SIMPLE IRA and a SIMPLE 401(k). All of the following statements apply to both types of SIMPLE plans except

The answer is there is a 25% penalty for early distributions from a participant's account within two years of entry into the plan. Only early distributions from a SIMPLE IRA within the first two years of initial participation in the plan are subject to the 25% early withdrawal penalty tax.

Larry is a sole proprietor of a business with 15 employees. He would like to implement a formal retirement plan for his business. Larry is 55 years old and is planning to retire in 10 years at age 65. His company currently has a strong cash flow, which is expected to continue. Larry's own personal savings retirement need is $85,000 per year, and he pays himself $95,000 annually. The company can afford to contribute $100,000 this year for Larry's account to any retirement plan that is implemented. Larry will also commit to an annual contribution necessary to fund the retirement plan if needed. Based on limited information, which of the following types of qualified retirement plans would you recommend for Larry and his business?

The answer is traditional defined benefit pension plan. This type of plan is most appropriate for Larry and his business. The business has favorable cash flow and can commit to the annual contribution required by the defined benefit approach. Additionally, Larry's savings need as a percentage of his compensation exceeds anything possible in a defined contribution plan. Finally, Larry is currently age 55 with only 10 years until retirement.

Required minimum distributions from a qualified plan to a plan participant must be calculated using the Uniform Lifetime Table in all cases except

The answer is when the designated beneficiary is the participant's spouse and the spouse is more than 10 years younger than the participant. The Uniform Lifetime Table must be used to calculate required minimum distributions under a qualified plan or IRA unless the designated beneficiary is the participant's spouse and the spouse is more than 10 years younger than the participant, in which case the actual joint life expectancy is used.

Your client, a widow, has made the following lifetime gifts to her son in an effort to reduce the size of her gross estate: YearGiftTaxable Gift2012$323,000$310,0002016$254,000$240,0002021$465,000$450,000 If she used her applicable credit to offset gift tax liability on the gifts, what amount of applicable credit remains available to your client for gifts in 2021?

The total taxable gifts come to $1,000,000 and tax on the first $1,000,000 is $345,800. Subtract that from the 2021 applicable credit of $4,625,800 and the result is $4,280,000.

In 2018, Roland established an inter vivos irrevocable trust naming his wife as the sole income beneficiary of the trust. All income must be distributed annually. At his wife's death, the balance of the trust will be in her gross estate. The trust was funded with $3 million in cash and assets. Which one of the following most closely approximates Roland's taxable gift for this transfer?

The unlimited marital deduction eliminates any gift tax liability on such a transfer.


Conjuntos de estudio relacionados

International Business - Chapter 5 - Trading Internationally

View Set

Texas Principles of Real Estate 1 (Exam Prep)

View Set

HITT 1311 - Study Questions / Check Your Understanding - Chapter 17

View Set

Physiological Psychology: Chapter 1

View Set